Administrative Law and Human Rights SBAQs Flashcards

You may prefer our related Brainscape-certified flashcards:
1
Q

Which of the following statements is correct in relation to Article 15 of the ECHR?

A. Under Article 15 of the ECHR, a State may derogate from part of the Convention as long as such derogation is proportionate.

B. Under Article 15 of the ECHR, a State may derogate from certain Articles of the Convention “in time of war or other emergency threatening the life of the nation.”

C. Under Article 15 of the ECHR, a State may derogate from any of the Articles of the Convention “in time of war or other emergency threatening the life of the nation.”

D. Under Article 15 of the ECHR, a State can derogate from certain Articles of the Convention because it is allowed “a margin of appreciation”.

A

Option B is correct

It is possible to derogate from some, but not all of the Articles of the Convention in time of war or other emergency threatening the life of the nation.

How well did you know this?
1
Not at all
2
3
4
5
Perfectly
2
Q

Which of the following statements is WRONG in relation to the European Court of Human Rights (‘ECtHR’)?

A. The ECtHR may reinterpret the ECHR over time as it is a ‘living instrument’.

B. The Committee of Ministers of the Council of Europe is responsible for ensuring that the judgments of the ECtHR are complied with by the signatory states.

C. The decisions of the ECtHR have direct binding force in the signatory states.

D. Cases may be brought before the ECtHR either on an inter-state basis or by individual application.

A

Option C is correct (I think)

Decisions of the ECtHR are only binding as a matter of international law and have no direct binding force in domestic law.

How well did you know this?
1
Not at all
2
3
4
5
Perfectly
3
Q

Which ONE of the following statements is CORRECT?

A. Under section 8 of the Human Rights Act, a court must award damages to the victim of an unlawful act.

B. Under section 8 of the Human Rights Act, a court may award damages to the victim of an unlawful act but only if that victim has suffered physical injury or monetary loss.

C. Under section 8 of the Human Rights Act, a court must award damages to the victim of an unlawful act if the court is satisfied that the award is necessary to afford just satisfaction to the victim.

D. Under section 8 of the Human Rights Act, a court may award damages to the victim of an unlawful act if the court is satisfied that the award is necessary to afford just satisfaction to the victim.

A

Option D is correct

Section 8 of the Human Rights Act enables a court to grant any relief or remedy within its powers. This will include the power to award damages, but such damages may be awarded only if necessary to afford just satisfaction to the victim of an unlawful act. An award of damages under s 8 is discretionary.

How well did you know this?
1
Not at all
2
3
4
5
Perfectly
4
Q

Which one of the following statements is WRONG?

A. Deprivation of life will not be in breach of Article 2 if it results from the use of force which is no more than absolutely necessary to defend any person from unlawful violence.

B. Deprivation of life will not be in breach of Article 2 if it results from the use of force which is no more than absolutely necessary to prevent the destruction of property of historical, cultural or religious significance.

C. Deprivation of life will not be in breach of Article 2 if it results from the use of force which is no more than absolutely necessary to effect a lawful arrest or prevent the escape of a person lawfully detained.

D. Deprivation of life will not be in breach of Article 2 if it results from the use of force which is no more than absolutely necessary to lawfully quell a riot or insurrection.

A

Option B is correcr

Article 2(2) provides that deprivation of life which results from the use of force which is no more than absolutely necessary will not contravene Article 2 if it is for the one of the reasons set out in A, C or D.

The Article makes no reference to the destruction of property of historical, cultural or religious significance.

How well did you know this?
1
Not at all
2
3
4
5
Perfectly
5
Q

Which of the following statements is correct?

A. Article 3 imposes a qualified duty on the state not to deport a person who would face a real risk of being subjected to treatment contrary to Article 3 in the receiving state.

B. Article 3 imposes an absolute duty on the state not to deport a person who would face a real risk of being subjected to treatment contrary to Article 3 in the receiving state.

C. Article 3 imposes no duty on the state not to deport a person who would face a real risk of being subjected to treatment contrary to Article 3 in the receiving state.

D. Article 3 does not apply to deportation cases if the person to be deported is a suspected terrorist.

A

Option B is correct

There are no circumstances in which a state could deport an individual and not be in breach of Article 3, if there is a real risk that the individual would be subjected to treatment contrary to Article 3 in the receiving state (Chahal v United Kingdom).

How well did you know this?
1
Not at all
2
3
4
5
Perfectly
6
Q

Which one of the following statements is correct?

A. Compulsory military service amounts to forced or compulsory labour.

B. Servitude is when someone is owned by another person.

C. Work done by convicted prisoners as part of their sentence does not amount to forced or compulsory labour.

D. An individual can be subject to forced labour even if they are not exploited.

A

There might be a mistake in the question but I don’t think any of these are correct

compulsory military service is not forced labour and neither is work done by convicted prisoners as part of their sentence. It is an essential element of forced labour that an individual is exploited. Slavery is when someone is owned by another; servitude includes an obligation for the serf to live on the property of another.

How well did you know this?
1
Not at all
2
3
4
5
Perfectly
7
Q

Standing at a bus stop, a man who is carrying a heavy rucksack is watched by armed security officers who have been instructed that he is a terrorist about to blow up a bus. As the bus pulls up, the security officers shout to stop the man from boarding bus but he ignores them. The security officers shout for a second time and, when ignored again, fire shots into the man who dies instantly. On checking the man’s body, they discover that he is wearing earphones and his rucksack is full of University library books.

The man’s parents believe that the government has breached their son’s Article 2 ECHR right to life. Are they able to take the government to court?

A. No, because the security officers acted reasonably in the circumstances. They believed that he was taking a bomb onto the bus and he failed to follow two orders to stop.

B. No, because only a person who is directly affected by the action of a public authority may claim a breach of their human rights.

C. No, because only the Crown Prosecution Service can decide to indict on a murder charge and are unlikely to do so since it was a matter of national security.

D. Yes, because a breach of Article 2 results in the death of the victim who cannot then take the matter to court personally. Next of kin, the parents here, can launch a challenge.

E. Yes, because they can write immediately to the European Court of Human Rights and ask for prosecution of the UK government for breaching their son’s Article 2 right to life.

A

The correct answer is D

Although only a directly affected victim can claim a breach of rights, an exception is made for Article 2 where the victim is dead. The next of kin may do so in his place.

A is wrong because it describes a defence to any such action and not whether the action may be taken in the first place.

B is wrong because the victim of a breach of Article2 rights is dead. It would be unfair if no challenge could be made by anyone else.

C is wrong because the parents do have the option to take the matter to court. The Crown Prosecution Service does, however, have the right to indict as well if they choose to do so.

E is wrong, because although the parents do have the right to approach the European Court of Human Rights directly, they cannot do so immediately. They must exhaust domestic remedies first.

How well did you know this?
1
Not at all
2
3
4
5
Perfectly
8
Q

An Egyptian citizen fled to the UK when he discovered that he was wanted in Egypt for taking part in a political demonstration. Whilst in the UK he was convicted of armed robbery. The UK government now wishes to deport him. The Egyptian citizen has evidence to show that if returned to Egypt he will be arrested and tortured for his part in the political demonstration and there is a real risk that he would be executed.

What is the Egyptian citizen’s legal position with respect to rights under the European Convention of Human Rights (‘ECHR’)?

A. He will be unable to rely on his rights under the ECHR as he is a citizen of a country which is not a signatory to that treaty.

B. He will be able to rely on his rights under Article 2 ECHR as his life is at real risk and Article 3 ECHR as he will be tortured if returned to Egypt.

C. He will be unable to rely on his rights under the ECHR as he has been convicted of a serious offence.

D. He will be able to rely on Article 2 alone because anything less than a real risk of death would not be a defence to deportation for such a serious offence.

E. He will be able to rely on Article 3 alone as it is not certain that he would be executed.

A

B is the correct answer - the Egyptian citizen will be able to rely on his rights under Article 2 ECHR as his life is at real risk and Article 3 ECHR as he will be tortured if returned to Egypt.

A and C are wrong as protections within the ECHR apply to all people within the jurisdiction of the UK, regardless of their nationality and whatever their circumstances.

D is wrong as Article 3 could also be relied on in these circumstances (Soering v UK)

E is wrong as it is enough that there is a ‘real risk’ of the death penalty – it does not need to be certain.

How well did you know this?
1
Not at all
2
3
4
5
Perfectly
9
Q

A country which is not a signatory to the European Convention on Human Rights (‘the Convention’) has asked the UK Government to extradite a woman living in the UK to stand trial for murder in that country. The woman holds the nationality of the country requesting her extradition, but not of any other country. The Secretary of State has ordered the woman’s extradition and the woman has appealed to the High Court against the extradition order. During the hearing she produces evidence that she could face the death penalty if extradited and the High Court accepts the woman’s evidence.

Which of the following best explains whether the High Court would uphold the extradition order?

A. It would not uphold the order because the Convention prohibits extradition to non-signatory countries.

B. It would not uphold the order because extraditing a person to stand trial for an offence which could result in the imposition of the death penalty would violate that person’s Convention rights.

C. It would not uphold the order because extraditing a person to stand trial for an offence which could result in the imposition of the death penalty fails to strike a fair balance between the rights of the individual and the interests of the community.

D. It would uphold the order because it is not certain that the death penalty would be imposed on the woman if she was extradited.

E. It would uphold the order because the Convention only protects the rights of people who hold the nationality of a state that has signed the Convention.

A

B is correct

Extraditing a person to a country where they could face the penalty would breach Article 2 (right to life) and probably also Article 3 of the Convention (prohibition of torture) (Soering and Article 1 of the 13th Protocol (abolition of the death penalty).

Option A is wrong because the Convention does not prohibit extradition to non-signatory countries.

Option C is wrong as the right to life and prohibition of torture are absolute rights and, unlike qualified rights, do not involve a balancing act between the rights of the individual and the interest of the community.

Option D is wrong as it is sufficient to engage Articles 2 and 3 if there is a possibility of the death penalty being imposed.

Option E is wrong as the Convention covers nationals of non-signatory states resident in signatory states.

How well did you know this?
1
Not at all
2
3
4
5
Perfectly
10
Q

In response to a spate of terrorist bombings in the UK, Parliament passes an Act of Parliament which grants emergency powers to the Minister for Anti-Terrorism allowing him to take ‘such steps as are deemed necessary’ to deal with the emergency. The security services have detained a suspect who they are sure knows the identity of the bombers. They ask the Minister to authorise enhanced interrogation of the suspect to include sleep deprivation and the holding of stress positions. Acting under his statutory powers the Minister authorises the use of enhanced interrogation.

Will the Minister be in breach of the European Convention on Human Rights (‘ECHR’)?

A. No, because the terrorist situation amounts to a public emergency threatening the life of the nation and so the UK can derogate from the ECHR.

B. No, because he has statutory authority for his actions.

C. Yes, because Article 15 only allows derogation from the ECHR where this is strictly necessary which it would not be on the facts.

D. Yes, because it is not possible to derogate from Article 3 and the enhanced interrogation techniques would amount to torture.

E. Yes, because it is not possible to derogate from Article 3 and the enhanced interrogation techniques would amount to inhuman or degrading treatment.

A

The correct answer is E

– the enhanced interrogation techniques would amount to inhuman or degrading treatment (Ireland v UK). It is not possible to derogate from Article 3.

Although the terrorist situation is likely to amount to a public emergency threatening the life of the nation, no derogation is possible from Article 3, so A is wrong.

The fact that the Minister has statutory authority for his actions will not prevent him from being in breach of the ECHR, so B is wrong.
The UK may be able to argue that the action it took is ‘strictly necessary’, but it is unnecessary to decide this, since no derogation is possible, so C is wrong.

The enhanced interrogation techniques will not amount to torture so D is wrong (Ireland v UK).

How well did you know this?
1
Not at all
2
3
4
5
Perfectly
11
Q

12 months ago a French national was arrested and charged with a terrorism offence in England. He claims that, whilst in detention, he suffered inhuman and degrading treatment at the hands of the security services. He brings a claim for breach of his rights under Article 3 of the European Convention on Human Rights (‘ECHR’) to the European Court of Human Rights (‘ECtHR’).

Will any claim the French national brings to the ECtHR be ruled admissible?

A. No, the claim will be ruled inadmissible because any proceedings will need to be brought by France as he is a French national.

B. No, the claim will be ruled inadmissible because any proceedings must be brought within four months of the decision being complained of.

C. No, the claim will be ruled inadmissible because he will first need to bring a claim before the UK courts.

D. Yes, the claim will be ruled admissible because the UK has breached an absolute right under Article 3 of the ECHR.

E. Yes, the claim will be ruled admissible because he is a victim of a breach of his rights under Article 3 ECHR.

A

C is correct

An individual must exhaust all domestic remedies before they can bring a claim to the ECtHR, so it will be clear that the claim will be inadmissible from the outset.

Option A is wrong as, although France may be able to bring a claim in these circumstances, this does not prevent an individual petition.

Option B is wrong as, although any claim must be brought within four months, the time limit will run from the final court decision within the UK, not the original decision.

Option D is wrong as, even if the UK has breached an absolute right, the French national must follow the correct procedure or the claim will be inadmissible.

Similarly, option E is wrong. Although the French national may be a victim, his claim will be inadmissible unless he first exhausts his domestic remedies.

How well did you know this?
1
Not at all
2
3
4
5
Perfectly
12
Q

A woman has been told that she needs to undertake jury service. She is allocated to a complex trial and so will be away from work for several months. Although she will receive expenses and can claim a small amount for loss of earnings from the court, this does not compensate for her loss of wages which are far higher. She claims that this breaches her rights under Article 4 of the European Convention on Human Rights (‘ECHR’).

Which of the following best represents the legal position under Article 4 ECHR?

A. There is no breach of Article 4 ECHR as the jury service does not amount to slavery.

B. There is a breach of Article 4 ECHR as the jury service will amount to forced or compulsory labour.

C. There is no breach of Article 4 as, although jury service amounts to forced or compulsory labour, this is a proportionate interference by the state with her Article 4 rights.

D. There is no breach of Article 4 ECHR as the jury service will not amount to forced or compulsory labour as it forms part of normal civic obligations.

E. There is no breach of Article 4 as the level of hardship caused by the jury service is insufficient to amount to forced or compulsory labour.

A

The correct answer is D. Work which forms part of normal civic obligations does not constitute forced or compulsory labour (Article 4(3) ECHR). Jury service would be part of normal civic obligations.

A is wrong as Article 4 is wider than slavery and also covers forced and compulsory labour.

B is wrong as there are some forms of compulsory work or service which are expressly stated not to amount to ‘forced or compulsory labour’ within Article 4, including normal civic obligations.

C is wrong as Article 4 is an absolute right and so cannot be interfered with, even where this is proportionate.

E is wrong as, although the level of hardship here may be insufficient to amount to forced or compulsory labour, this is debateable. Jury service is clearly a ‘normal civic obligation’ and so D represents the better explanation of the legal position.

How well did you know this?
1
Not at all
2
3
4
5
Perfectly
13
Q

A man is shot and killed by the police. His mother was dissatisfied with the investigation into the death that followed. She brought a claim for breach of Article 2 ECHR, but her claim failed before the UK courts. She then applied to the European Court of Human Rights (‘ECtHR’) and her claim was heard by a Chamber of judges who gave judgment in her favour last week.

Which of the following statements is correct in relation to the decision of the ECtHR?

A. The decision of the ECtHR is binding on the UK.

B. The mother will be able to enforce the decision of the ECtHR in the UK courts.

C. The ECtHR will ensure that the UK complies with its ruling.

D. The decision of the Chamber of judges is final and the UK government can take no further action in relation to it in the ECtHR.

E. The decision of the ECtHR will be binding on future cases brought before the ECtHR.

A

The correct answer is A.

Judgments of the ECtHR are binding on the UK as a matter of international law.

Option B is wrong as, although decisions of the ECtHR are binding in international law, they have no direct binding force in domestic law.

Option C is wrong as the Committee of Ministers of the Council of Europe is responsible for ensuring compliance rather than the court.

Option D is wrong as the UK will be able to ask for the case to be heard before the Grand Chamber for up to three months after the decision has been made by the Chamber of judges.

Option E is wrong as the ECtHR does not operate a system of binding precedent. The ECHR is a ‘living instrument’ so that the ECtHR can reinterpret its provisions over time.

How well did you know this?
1
Not at all
2
3
4
5
Perfectly
14
Q

Which of the following is NOT a requirement under Article 5 of the ECHR?

A. The detained person must be brought before a court promptly.

B. The detained person must be allowed to inform relatives of their whereabouts within a reasonable time.

C. The detained person must be able to challenge the legality of their detention.

D. The detained person has a right to compensation for any breach of Article 5.

A

B is not a requirement under article 5 of the ECHR

Answers A,C and D are all requirements set out in Article 5, but these requirements do not include that the detained person must be able to inform their relatives.

How well did you know this?
1
Not at all
2
3
4
5
Perfectly
15
Q

Jane is bringing a claim for breach of contract against KitchenKraft as she claims the units they supplied were defective.

Is the following statement true or false?

Article 6 provides that Jane must have a fair hearing.

A

The statement is true

A breach of contract claim will involve ‘civil rights and obligations’ and so Jane will be entitled under Article 6 to a fair hearing.

How well did you know this?
1
Not at all
2
3
4
5
Perfectly
16
Q

Which of the following statements is INCORRECT?

A. Article 6(1) of the European Convention on Human Rights states that the press and public may be excluded from all or part of a trial in the interests of public order.

B. Article 6(1) of the European Convention on Human Rights states that the press and public may be excluded from all or part of a trial in the interests of the economic well-being of the country.

C. Article 6(1) of the European Convention on Human Rights states that the press and public may be excluded from all or part of a trial in the interests of juveniles.

D. Article 6(1) of the European Convention on Human Rights states that the press and public may be excluded from all or part of a trial in the interests of national security.

A

Answer B is incorrect

Article 6(1) allows a court to exclude the press and public from all or part of a trial for several reasons, which include the interests public order, juveniles or national security. It contains no reference to the economic well-being of the country.

How well did you know this?
1
Not at all
2
3
4
5
Perfectly
17
Q

Article 6 of the ECHR states that an individual charged with a criminal offence is entitled to free legal assistance in which of the following circumstances?

A. If the individual has not sufficient means to pay for legal assistance and he may lose his liberty if convicted.

B. If the individual has not sufficient means to pay for legal assistance and the interests of justice so require.

C. If the individual is to be tried before the Crown Court.

D. If the individual is not in gainful employment.

A

Answer B is correct

An individual must be provided with free legal assistance if he does not have sufficient means to pay for this, and the interests of justice so require.

How well did you know this?
1
Not at all
2
3
4
5
Perfectly
18
Q

Mo has been charged with a strict liability offence. He argues that he did not intend to commit the offence and that to convict him would breach Article 6 ECHR. Which of the following statements is true?

A. It would breach Article 6 to convict Mo of a strict liability offence.

B. It would not breach Article 6 to convict Mo of a strict liability offence provided the offence was serious.

C. It would not breach Article 6 to convict Mo of a strict liability offence provided that Mo had received legal advice and representation.

D. It would not breach Article 6 to convict Mo of a strict liability offence provided that this was reasonable.

A

Answer D is correct

Strict liability offences do not automatically breach Article 6 and the presumption of innocence. However, such offences must be reasonable (Salabiaku v France).

How well did you know this?
1
Not at all
2
3
4
5
Perfectly
19
Q

Contempt of Court may be either civil contempt or criminal contempt.

Is the following statement true or false:

It is an example of civil contempt of court to publish an article which interferes with the course of justice.

A

The statement is false

It is contempt of court to publish an article which interferes with the course of justice, but this is a criminal offence not a civil matter.

How well did you know this?
1
Not at all
2
3
4
5
Perfectly
19
Q

The Caged Bird Act 2010 (fictitious) states that it is an offence punishable by a fine of up to £1000 to keep a bird in a cage. Two months ago Jay was charged with an offence of keeping a bird in a cage. A month ago the fine for an offence under the Caged Bird Act was increased to £5000. Jay appears in court today, is convicted and receives a fine of £2500.

Is the following statement true or false:

The fine Jay receives breaches Article 7 ECHR

A

The Statement is true

Jay would be receiving a heavier penalty than was applicable at the time he committed the offence two months ago (when he could only have received a maximum fine of £1000). This breaches Article 7.

How well did you know this?
1
Not at all
2
3
4
5
Perfectly
20
Q

A robbery takes place. The police wish to interview a man who they know has witnessed the robbery and may be able to identify the participants. The man is unwilling to answer police questions, but is told that he must come to the police station or he will be arrested. The police escort the man to the station where he is briefly questioned before being escorted to the visitors’ lounge. He is given refreshments and told he cannot leave until the police have finished questioning him. He is interviewed seven hours later. He tells the police everything he knows and is then released. The man applies to court claiming his right to liberty and security under Article 5 of the European Convention on Human Rights (ECHR) has been breached.

Which of the following statements best explains whether a court will find that the man’s right to liberty under Article 5 ECHR has been breached?

A. The court will find that the man was not deprived of his liberty as he was not placed in a police cell at any time.

B. The court will find that the man was not deprived of his liberty as he was detained at the police station for less than 24 hours.

C. The court will find that the man was not deprived of his liberty as the actions of the police were lawful and proportionate.

D.The court is likely to find that the man has been deprived of his liberty and this deprivation was not within the limitations permitted under Article 5.

E. The court is likely to find that the man has been deprived of his liberty but this deprivation was within the limitations permitted under Article 5.

A

The correct answer is D

The court is likely to find that the man has been deprived of his liberty as he has been coerced into going to the police station and held there for a significant period of time.

The detention does not fall within any of the six limitations in Article 5(1) since he has not been convicted of an offence; has not been arrested on suspicion of an offence, to prevent him committing an offence or to comply with a court order; is not a minor or mentally ill and being detained for protection and the case has nothing to do with asylum deportation, or extradition – so E is wrong.

A is wrong as it is not necessary for someone to be placed in a cell to be deprived of their liberty (Guzzardi).

There is no strict definition of ‘deprivation of liberty’ so that, although duration will be taken into account, there is no rule that it needs to be for at least 24 hours (Austin v UK) so B is wrong.

C is wrong as the actions of the police are not lawful as the man was not detained under due process of law.

How well did you know this?
1
Not at all
2
3
4
5
Perfectly
20
Q

Article 5 of the European Convention on Human Rights provides a right to security and liberty of the person. However, Article 5 states that people can be lawfully deprived of their liberty in certain circumstances.

Which of the following is NOT a lawful reason for the deprivation of liberty under Article 5(1) of the ECHR?

A. The lawful detention of a person after conviction by a competent court.

B. The lawful arrest or detention of a person for the purpose of bringing them before the competent legal authority on reasonable suspicion of having committed an offence.

C. The lawful arrest or detention of a person for non-compliance with the lawful order of a court.

D. The lawful detention of a person for the purpose of ensuring they have necessary medical treatment which they have refused to have.

E. The lawful detention of a person in order to deport them.

A

Option D is the correct answer

Answers A, B, C and E are listed in Article 5(1) as lawful limitations on the right to liberty.

Article 5(1)(e) does permit the lawful detention of persons for the prevention of the spreading of infectious diseases, but there is no wider requirement which permits detention for the purpose of necessary medical treatment which the person has refused to have.

How well did you know this?
1
Not at all
2
3
4
5
Perfectly
21
Q

A young woman has been murdered. The police suspect her stepfather may be involved in her murder and he is arrested and taken to the police station for questioning. Several national newspapers publish articles vilifying the young woman’s stepfather and stating that the police had ‘got their man’. When questioned at a news conference the police commented ‘this is a complex case and we wouldn’t anticipate that any trial would take place for at least six months’. The Attorney General is contemplating taking proceedings for contempt of court against the national newspapers.

Would the Attorney General be able to commence proceedings for contempt of court under the Contempt of Court Act 1981?

A. Yes, because proceedings are active as the stepfather is under police investigation.

B. Yes, because proceedings are active as the stepfather has been arrested.

C. No, because proceedings are not active as the date of trial is at least six months away and so potential jurors will not remember the contents of the article.

D. No, because proceedings are not active as the stepfather has not been charged with a criminal offence.

E. No, because proceedings are not active as the case has not been set down for trial and the date of the trial has not been fixed.

A

B is the correct answer.

The proceedings are criminal proceedings (for murder) and so they will become active on arrest (s2(3) and schedule 1 Contempt of Court Act), rather than the commencement of the police investigation (A is wrong) or charge (D is wrong).

C is wrong as the length of time before trial may be relevant to whether the article caused a ‘substantial risk’ of prejudice under s2(2) Contempt of Court Act, but not to whether proceedings are active.

E is wrong as these are the time limits for civil proceedings.

How well did you know this?
1
Not at all
2
3
4
5
Perfectly
22
Q

The police receive an anonymous tip-off that Bert is a member of an armed gang which recently stole a valuable Picasso painting. At noon on Tuesday PC Jones sees Bert walking towards his home and arrests him. At the police station the custody sergeant tells Bert that he is being detained without charge as this is necessary to obtain evidence of the offence by questioning him. Bert is interviewed and makes no comment to all questions asked by the police. He is then placed in a cell. The police spend the day investigating the armed robbery but do not interview Bert further. On Wednesday morning the custody sergeant authorises Bert’s continued detention. It is now Thursday.

Which of the following statements best represents whether Bert is lawfully detained by the police?

A. Bert is being detained unlawfully as the police did not have a reasonable suspicion that he was involved in the theft of the painting.

B. Bert is being detained unlawfully as he has been detained for more than 24 hours.

C. Bert is being detained unlawfully as he has been detained for over 24 hours without this being authorised by a police officer of the rank of superintendent or above.

D. Bert is being detained lawfully as the offence is an indictable one and the investigation is being carried out diligently and expeditiously.

E. Bert is being detained lawfully as the maximum length of detention is 96 hours when the offence is an indictable one.

A

The correct answer is C

Detention for over 24 hours (up to 36 hours) must be authorised by a police officer of the rank of superintendent or above. A custody sergeant is not of sufficient rank.

A is wrong as, although it may be debateable whether the police have a reasonable suspicion, they may have, and so this is not the best answer.

B is wrong as suspects can be detained for over 24 hours provided that certain conditions are fulfilled.

D is wrong as, although the offence is an indictable one and it may be being investigated diligently and expeditiously, this is uncertain and, in any event, the correct authorisation has not been given.

E is wrong as, although the maximum length of detention is 96 hours where certain conditions are fulfilled, including that the offence is an indictable one, this on its own is not sufficient.

How well did you know this?
1
Not at all
2
3
4
5
Perfectly
23
Q

A man is under arrest for suspected theft of jewellery. While he is at the police station the police search his girlfriend’s property using a search warrant. The search warrant has been obtained improperly and they have no power to carry out the search. During the search the police discover some jewellery which matches the description of the stolen property. The man is charged with theft of the jewellery.

Can the jewellery be given in evidence at the man’s trial?

A. No, because the search of the girlfriend’s property was unlawful.

B. No, because the jewellery was found on the girlfriend’s property rather than on the man’s property.

C. No, because to do so would mean that the man would not have a fair trial.

D. Yes, because it is irrelevant that the search warrant was obtained improperly.

E. Yes, because the jewellery is relevant evidence and there is no reason to question its reliability.

A

E is the correct answer

Evidence that is unlawfully obtained is still admissible provided that it is relevant to an issue and is not unreliable, so that the defendant will still have a fair trial (Khan v UK).

As the trial will be deemed to be fair, C is wrong.

Although the search of the property was unlawful, this alone will not mean the evidence is inadmissible so A is wrong.

B is wrong as it is irrelevant where the jewellery was found as long as it is relevant evidence.

D is wrong as it is not irrelevant that the evidence was obtained improperly, however this fact alone will not lead to such evidence being ruled inadmissible.

How well did you know this?
1
Not at all
2
3
4
5
Perfectly
24
Q

A woman has been arrested for theft. She is a drug addict and soon begins to suffer withdrawal symptoms and to feel very unwell. She tells this to the police who reply “well, if you admit you stole the goods, we can release you straightaway”. The woman confesses to the theft and is released. The police intend to use the woman’s confession at trial. The woman argues that this would breach her right to a fair trial under Article 6 of the European Convention on Human Rights.

Which of the following best reflects whether the woman’s confession will be excluded from the evidence given at trial?

A. The woman’s confession will not be excluded as it was freely given.

B. The woman’s confession will be excluded as she is a drug addict and therefore a vulnerable person.

C. The woman’s confession will not be excluded as the police have not acted oppressively.

D. The woman’s confession will be excluded as it is likely to be unreliable.

E. The woman’s confession may be excluded as it is likely to be unreliable.

A

The correct answer is D.

The confession will be excluded as it is likely to be unreliable as the woman is suffering withdrawal symptoms and has received an inducement to confess by being told she will be released (s76 PACE).

A is wrong as it is arguable whether the confession is ‘freely given’ and, even if it is, under the circumstances it is likely to be unreliable as explained above.

B is wrong as the crucial question is whether the confession is unreliable rather than whether the woman is a vulnerable person.

C is wrong as, under s76 PACE it is not necessary for the police to have acted oppressively.

E is wrong as the court is under a duty to exclude unreliable evidence, it does not have discretion.

How well did you know this?
1
Not at all
2
3
4
5
Perfectly
25
Q

A woman is arrested on suspicion of theft of a debit card. She was arrested after a passer-by reported to the police that a woman was acting suspiciously at a cashpoint machine. The card the woman had inserted into the cashpoint machine did not belong to the woman. At the police station the woman was offered legal advice but refused this and gave a ‘no comment’ interview. At court the woman explained that the card belonged to her friend who had asked her to withdraw some money for her. The police have evidence that the friend had reported the card as stolen, although had subsequently supported the woman’s story.

Will the court be able to draw adverse inferences from the woman’s failure to account for having the card in her possession when interviewed by the police?

A. Yes, because adverse inferences can be drawn from the woman failing to account for having the debit card in her possession when questioned by the police as theft is an indictable offence.

B. Yes, because adverse inferences can be drawn from the woman failing to account for having the debit card in her possession when questioned by the police as the police have other evidence which implicates her.

C. No, because adverse inferences cannot be drawn from an arrested person remaining silent when questioned about an offence by the police.

D. No, because adverse inferences cannot be drawn since the woman has a cogent explanation for having the card in her possession.

E. No, because adverse inferences cannot be drawn from the woman failing to account for having the debit card in her possession when questioned by the police as she had not received legal advice.

A

B is the correct answer

Adverse inferences can be drawn by the court when a defendant fails to account for an object in their possession when asked to do so by the police provided that this would not lead to them being convicted on the basis of this evidence alone (Murray v UK).

A is wrong because it is irrelevant that the offence is an indictable one.

C and D are wrong as adverse inferences can be drawn - the inference being that at the police station the woman did not have an innocent explanation as to why she had the debit card (and this was concocted (with the friend) at a later date).

Adverse inferences can be drawn where the defendant has not received legal advice, as long as she has been offered such advice, so E is wrong.

How well did you know this?
1
Not at all
2
3
4
5
Perfectly
26
Q

Flora is a committed ‘anti-vaxxer’ who has been arrested after violence broke out at a recent protest outside a vaccination clinic. She was subsequently charged with a serious public order offence and is due to be tried at Notown Crown Court next week. Gordon is the editor of Daybreak News, a national newspaper. Daybreak News has published a series of editorials criticising anti-vaxxers. Today, Daybreak News published an editorial which contained the following:

‘The desperate Anti-vaxxers have now turned to violence to further their cause. Flora Foy, a well-known Anti-Vaxxer is due on trial next week after participating in the violence at the Notown vaccination clinic. Daybreak readers, we call on you to do your duty and ensure that Flora is convicted so that she will hopefully spend some time in prison where she can contemplate the folly of her anti-vaxxer beliefs.’

Which of the following statements best explains Gordon’s potential liability if he is prosecuted for contempt of court?

A. Gordon can be found guilty of contempt of court under the Contempt of Court Act 1981 under the strict liability rule embodied in the Act as any contempt is unintentional.

B. Gordon can be found guilty of contempt of court under the Contempt of Court Act 1981 under the strict liability rule embodied in the Act as the Contempt of Court Act abolished common law contempt.

C. Gordon can be found guilty of contempt of court under the Contempt of Court Act 1981 under the strict liability rule and at common law for intentional contempt of court.

D. Gordon can be found guilty under the Contempt of Court Act 1981 as the Contempt of Court Act 1981 makes intentional contempt of court a criminal offence.

E. Gordon can be found guilty of contempt of court under the Contempt of Court Act 1981 and at common law for unintentional contempt of court.

A

The correct answer is C.

The article in Daybreak News appears to be an intentional attempt to influence potential jurors to find Flora guilty and thus interfere with the administration of justice which would be an offence at common law and A is wrong.

The article will create a ‘substantial risk of serious prejudice’ to the proceedings which are active (Flora has been arrested and charged). None of the defences in the Contempt of Court Act would apply and the risk of prejudice is not merely incidental to a general discussion of matters of public interest so s5 will not apply. The offence under the Contempt of Court Act has also been committed.

B is wrong as the Contempt of Court Act did not abolish the common law offence of intentional contempt.

D is wrong as the Contempt of Court Act makes unintentional contempt an offence, not intentional contempt, which is an offence at common law.

E is wrong for both common law contempt (which is intentional) and contempt under the Contempt of Court Act which covers unintentional contempt.

How well did you know this?
1
Not at all
2
3
4
5
Perfectly
27
Q

Internal guidelines allow local authorities to ‘take such action as they see fit’ to protect children fostered by them. Acting under these guidelines a local authority seizes the laptops of a couple who have been foster carers for the local authority for many years. The local authority finds that one of the couple has been regularly visiting pornography sites featuring adults only. They email the foster carers stating that they are no longer approved as carers and will not be used by the local authority in future.

Have the local authority breached the Article 8 ECHR rights of the foster couple?

A. No, because the local authority have acted to protect the rights of children who are vulnerable persons so the local authority action is proportionate.

B. Yes, because the pornography sites feature adults not children and so the local authority have acted in a disproportionate manner.

C. Yes, because the local authority are acting under internally published guidelines and so not ‘in accordance with the law’.

D. No, because the local authority action is prescribed by law, has a legitimate aim and seizing the computer is a proportionate way to protect children.

E.Yes, because the local authority action is prescribed by law and has a legitimate aim, however refusing to use the couple as foster carers in the future is disproportionate.

A

C is the correct answer.

The local authority internal guidelines are not accessible to the foster parents and, even if they were, are not sufficiently precise to enable the foster parents to regulate their conduct based on them. The seizure of their computers is thus not ‘prescribed by law’.

A is wrong because, although the children are vulnerable persons, that alone will not mean any action taken by the local authority will be proportionate. Seizing the computers is for a legitimate aim, but it is not prescribed by law, so D and E are wrong.

The local authority may have acted disproportionately, although this is not clear on the facts, so B is wrong.

How well did you know this?
1
Not at all
2
3
4
5
Perfectly
28
Q

A civil servant was dismissed by a government department because she refused to hide or to remove a necklace and charm she was wearing. The civil servant explained that the charm was symbolic and highly significant to her religion. The explanation was rejected as irrelevant, and the dismissal was confirmed on the ground that the necklace and charm detracted from the department’s professional image. She was told that she should work elsewhere if it was so important to her.

Has the government department acted in a way that is compatible with the civil servant’s right under Article 9 European Convention on Human Rights?

A. Yes, the department has every right to protect its professional image and to insist that the civil servant remove or hide the necklace.

B. No, the civil servant has the right to wear anything that allows her to manifest her religion at work despite the department’s opposition.

C. No, the department cannot prevent the civil servant wearing the religious jewellery because that would interfere with an absolute right.

D. No, although the department does have the right to protect its professional image, it must also take account of the civil servant’s right to manifest her religion.

E. Yes, the civil servant must always abide by the department’s policy regarding jewellery and should work elsewhere if this is a problem.

A

D is the correct answer because the civil servant has the right to manifest her religion. The department does have the right to protect its professional image but would need to balance this against the Article 9 right.

A is wrong because the department cannot insist that the civil servant remove or hide a piece of religious jewellery without good reason.

B is wrong because there are circumstances in which a department might insist that a civil servant cannot wear something which manifests that person’s religion e.g. for safety reasons.

C is wrong because Article 9 is not an absolute right. It is a qualified right which a public body can restrict in the public interest.

E is wrong because a civil servant does have an Article 9 right to manifest religion which may take the form of jewellery. This right should not be restricted unless there is good reason to do so. Here she should not need to change jobs.

How well did you know this?
1
Not at all
2
3
4
5
Perfectly
29
Q

An Act of Parliament established a Commission to decide where incinerators to dispose of household waste shall be built in England and Wales. A woman has objected to a decision made by the Commission approving the building of an incinerator 200 metres from her home. Her main ground of objection is that emissions will harm her and other people living nearby. Experts had produced evidence to the Commission that there were other suitable sites in the locality further away from people’s houses.

Which of the following best describes whether the building of the incinerator breaches any of the woman’s Convention rights?

A. The building of the incinerator does not engage any Convention rights as it is in the public interest.

B. Although the building of the incinerator engages the woman’s right to a private life, it is a proportionate interference in the public interest.

C. Although the building of the incinerator engages the woman’s right to a private life, it is prescribed by law and so cannot be challenged.

D. Although the building of the incinerator is in the public interest, it is a disproportionate interference with the woman’s right to a private life.

E. The building of the incinerator is not prescribed by law and is a disproportionate interference with the woman’s right to a private life.

A

D is correct.

The woman may be able to argue that her Article 8 right (right to a private life) is being breached; accordingly option A is wrong as Article 8 is engaged. However, Article 8 is a qualified right. The qualification has a legal basis here - the Act of Parliament; hence option E is wrong.

The legitimate aim(s) being pursued by the Act are likely to be the ‘economic well-being of the country’. However, the qualification must also be ‘necessary in a democratic society’.

Option C is therefore wrong because it incorrectly suggests that it is sufficient if the interference is prescribed by law.
As the qualification has a legal basis and the Act is pursuing a legitimate aim, it is necessary to apply the proportionality test. In other words, is the interference with the woman’s rights proportionate to the objective being achieved, or would any lesser interference be possible

In this case, it seems likely that there are more suitable sites for the incinerator, away from residential areas. Option B is therefore wrong because it states the interference is proportionate.

How well did you know this?
1
Not at all
2
3
4
5
Perfectly
30
Q

A man gave a speech in the town square holding a placard. One side of the placard read ‘Gay rights are immoral!’. The other side read ‘Stop homosexuality! Stop lesbianism!’ Several people were upset by this and the man was arrested and convicted of an offence of displaying writing that is abusive causing harassment, alarm or distress under s5 Public Order Act 1986. The man challenged his conviction in the UK on the basis that it breached his rights under Article 10 ECHR but was unsuccessful. He now plans to take his case to the European Court of Human Rights (‘ECtHR’)

Which of the following best describes the likely judgment and reasoning of the ECtHR?

A. The man’s challenge would not be successful, because although the conviction interfered with the man’s rights under Article 10 it was lawful as it was prescribed by law, for a legitimate aim and proportionate.

B. The man’s challenge would be successful, because although the man’s sign was insulting, Article 10 protects speech which is offensive or disturbing and so the conviction was not a proportionate interference with the man’s freedom of expression.

C. The man’s challenge would be unsuccessful, because although the conviction interfered with the man’s Article 10 rights the ECtHR gives states a wide margin of appreciation in this area.

D. The man’s challenge would be successful, because although the man’s sign was insulting it represented political free speech and so would be given a high degree of protection meaning that the conviction would breach the man’s article 10 rights.

E. The man’s challenge would be unsuccessful, because the placard undermined others in a way that was incompatible with the values underpinning the European Convention on Human Rights such as respect and non-discrimination.

A

The correct answer is E.

The ECtHR in the similar case of Norwood v UK stated that Article 10 would not protect views that were incompatible with the values that underpinned the European Convention on Human Rights, such as tolerance, respect and non-discrimination.
The man’s application is likely to be ruled inadmissible and so the ECtHR would not go on to consider whether any interference was prescribed by law, for a legitimate aim or proportionate, thus A is wrong as it is not the best answer.

Although Article 10 protects speech that is offensive and gives high protection to political speech, this protection is not unlimited and would not cover expression that undermines the values underpinning the ECHR, so B and D are wrong.

This is not an area where the ECtHR give a wide margin of appreciation to states so C is wrong.

How well did you know this?
1
Not at all
2
3
4
5
Perfectly
31
Q

A group campaigning for a law ensuring that people do not need to sell their homes to pay for care in their old age wishes to promote their campaign with an advertisement on primetime television. The broadcasting authority refuses to give permission for this on the basis that the advertisement is political. The campaigning group claims that such a refusal breaches its rights under Article 10 of the European Convention on Human Rights (‘ECHR’).

Would the campaign group be successful if they brought a legal challenge against the refusal of the broadcasting authority based on Article 10 ECHR?

A. No, any challenge based on Article 10 ECHR will be unsuccessful as Article 10 does not cover commercial television broadcasts.

B. Yes, a challenge based on Article 10 ECHR would be successful as regulation of broadcasting is not a legitimate aim under Article 10.

C. No, a challenge based on Article 10 ECHR would be unsuccessful as the ban is proportionate since alternative means of getting the campaigning group’s message across are available.

D. Yes, a challenge based on Article 10 ECHR would be successful, as a blanket ban on political advertisements is disproportionate.

E. Yes, a challenge based on Article 10 would be successful because freedom of political expression is crucial.

A

the correct answer is C. The ban on political advertising was held to be proportionate and not in breach of article 10 in the case of Animal Defenders v UK, so D and E are wrong.

Article 10 covers all expression, including commercial expression so A is wrong.

The ban protects the rights of others as it preserves the impartiality of broadcasting and protects the democratic process (Animal Defenders), so B is wrong.

How well did you know this?
1
Not at all
2
3
4
5
Perfectly
32
Q

A local authority plans to build a ring road which will pass very close to the offices of a company. The new road will improve travelling times and, as the office is close to the road, office rental prices will be at a premium. The company directors complain that the new road will cause pollution and noise. The company wishes to challenge the local authority’s plan on the basis that building the road would interfere with its rights under Article 1 of the First Protocol to the European Convention on Human Rights – protection of property (‘Article 1, First Protocol’).

Will the company’s challenge be successful?

A. No, the company’s challenge will be unsuccessful as Article 1, First Protocol does not apply to commercial property.

B. No, the company’s challenge will be unsuccessful as the company has not suffered financial loss.

C. No, the company’s challenge will be unsuccessful as the company will not be deprived of its property.

D. Yes, the company’s challenge will be successful as the local authority does not have a legitimate aim for building the ring road.

E. Yes, the company’s challenge will be successful as building the ring road would be a disproportionate interference with the company’s peaceful enjoyment of its possessions.

A

The correct answer is B as Article 1 of the First Protocol will only be engaged where the interference affects the financial value of property.

A is wrong as Article 1, First Protocol applies to any property or possessions.

C is wrong as Article 1, First Protocol applies where interference restricts the use – ‘peaceful enjoyment’ of property or possessions as well as when someone is deprived of their property.

D and E are wrong as the challenge will be unsuccessful. In addition, it is likely to be in the public interest for the ring road to be built (ie a legitimate aim exists) and there is nothing to suggest that the local authority have acted disproportionately.

How well did you know this?
1
Not at all
2
3
4
5
Perfectly
33
Q

A migrant woman recently arrived in the United Kingdom illegally. She applied for residence in the UK, but this was refused, and the Home Secretary has now ordered her removal from the UK to her country of origin. The woman suffers from an eating disorder for which she has received treatment whilst in the UK. The woman is concerned that, if she is returned to her country of origin, her eating disorder will worsen. Treatment for eating disorders is available in her country of origin although the standard of service is lower than that provided in the UK. The woman wishes to challenge the Home Secretary’s decision on the basis that it violates her right to respect for her private life.

Is the woman likely to be successful in her challenge to the Home Secretary’s decision?

A. No, because as she has entered the UK illegally the woman will not be able to rely on Article 8 ECHR.

B. No, because any violation of the woman’s rights would be minimal and so Article 8 will not be engaged.

C. No, because the issue involved is not the severance of family or social ties and so Article 8 will not be engaged.

D. Yes, because health consequences of removal come within Article 8 and the woman’s health would be adversely affected by being returned to her country of origin.

E. Yes, because, although extradition of the woman would be in pursuit of a legitimate aim, returning the woman to her country of origin would be disproportionate given that she has an eating disorder.

A

The correct answer is B.

Article 8 may be engaged when the main issue is the consequence for physical or mental health of removal and is not restricted to severance of family or social ties so C is wrong.

However, the threshold for establishing this is high and the woman would need to show that the violation of her rights would be flagrant (R(Razghar) v Secretary of State for the Home Department), so D and E are wrong.

Anyone within the UK can rely on their rights under the ECHR, including those who have entered illegally, so A is wrong.

How well did you know this?
1
Not at all
2
3
4
5
Perfectly
34
Q

A man employed as a shop assistant in a clothing shop has a small fish symbol tattooed on to his hand. The fish symbol is a Christian symbol. The clothing shop’s employment policy permits shop assistants to have small tattoos that are visible to customers but prohibits tattoos which have religious significance as it wants to adopt a secular image. The shop has taken disciplinary action against the man. The UK courts have upheld the lawfulness of the disciplinary action, so the man now wants to take action against the UK Government before the European Court of Human Rights.

Can the man argue that the disciplinary action breaches his Convention rights?

A. Yes, because the shop’s refusal to allow him to display a small tattoo means that the state has permitted a disproportionate interference with his right to manifest his religion.

B. Yes, because the shop’s refusal to allow him to display a small tattoo means that the state has permitted an interference with his absolute right to manifest his religion.

C. Yes, because the shop’s refusal to allow him to display a small tattoo means that the state has permitted an interference with his absolute right to freedom of religion.

D. No, because the shop has treated all religious symbols equally, there is no interference with the man’s freedom to manifest his religion.

E. No, because the shop’s refusal to allow him to display a small tattoo is a proportionate interference with the man’s freedom to manifest his religion.

A

A is correct.

Based on the case of Eweida, it seems improbable that a small discreet religious symbol would detract from the image that the shop wants to project. It is therefore a disproportionate interference with the man’s qualified right to manifest his religious belief.

Option B is wrong because the right to manifest a religious belief is a qualified right, not an absolute right. Option C is wrong because having a tattoo comes within the scope of manifesting religious belief, a qualified right, rather than holding a religious belief, an absolute right.

Option D is wrong because treating all religions the same does not mean that the right to manifest one’s religion fails to be engaged.

Option E is wrong as the interference with the man’s right to manifest his religion seems to go further than necessary to maintain the shop’s image.

How well did you know this?
1
Not at all
2
3
4
5
Perfectly
34
Q

Is the following statement true or false?

A young man suffering from an eating disorder is forced to have medical treatment against his wishes. This will engage his rights under Article 8.

A

The statement is true

Article 8 covers a wide range of issues, including a right to bodily integrity.

How well did you know this?
1
Not at all
2
3
4
5
Perfectly
35
Q

Clodagh Brown is a receptionist for the East Blankshire Secular Society (‘EBSS’). Clodagh wants to wear a crucifix badge to work to express her Christian faith. The CEO tells her she cannot wear this to work since ‘this is an organisation working towards a secular society. It would undermine our aims and offend visitors for the receptionist to visibly display a religious symbol’. Clodagh decides to bring a court case on the basis that this amounts to indirect discrimination and breaches her rights under Article 9 ECHR.

Assume that the court accepts that Clodagh’s Article 9 ECHR right has been engaged. When deciding the case the court will need to be satisfied that any interference with Article 9 is justified.

Which one of the following arguments EBSS could raise has the best chance of successfully persuading the court that the interference was justified?

A. An argument that it was up to the EBSS, as an employer, to decide its own policy on the wearing of religious symbols, unless that policy was so unreasonable that no reasonable employer could adopt it.

B. An argument that the ban was a proportionate means of achieving a legitimate aim recognised by the ECHR.

C. An argument that Clodagh’s Article 9 rights are less important than the rights of the EBSS and those visiting the EBSS.

D. An argument that allowing Clodagh to wear the crucifix would conflict with the aims of the EBSS.

A

The correct answer is B.

Article 9 is a qualified right which can be interfered with provided that this is prescribed by law, for a legitimate aim and proportionate.

Option A represents the irrationality test used in ‘domestic’ grounds for judicial review.

The arguments in C and D may be taken into account as part of the proportionality exercise, but are less likely than B to succeed if argued alone.

How well did you know this?
1
Not at all
2
3
4
5
Perfectly
36
Q

Which one of the following statements about section 5 of the Public Order Act (1986) is CORRECT?

A. Under section 5, it is a criminal offence for someone to use threatening or abusive words or behaviour, or to display any writing, sign or other visible representation which is threatening or abusive within the hearing or sight of a person likely to be caused harassment, alarm or distress.

B. Under section 5, it is a criminal offence for someone to use offensive or abusive words or behaviour, or to display any writing, sign or other visible representation which is offensive or abusive within the hearing or sight of a person likely to be caused harassment, alarm or distress.

C. Under section 5, it is a criminal offence for someone to use racist or abusive words or behaviour, or to display any writing, sign or other visible representation which is racist or abusive within the hearing or sight of a person likely to be caused harassment, alarm or distress.

D. Under section 5, it is a criminal offence for someone to use offensive or racist words or behaviour, or to display any writing, sign or other visible representation which is offensive or racist within the hearing or sight of a person likely to be caused harassment, alarm or distress.

A

The correct answer is A. Under s 5 of the Public Order Act (1986), it is a criminal offence for someone to use threatening or abusive words or behaviour, or to display any writing, sign or other visible representation which is threatening or abusive within the hearing or sight of a person likely to be caused harassment, alarm or distress.

How well did you know this?
1
Not at all
2
3
4
5
Perfectly
37
Q

A well-known actor was photographed leaving a walk-in HIV testing clinic in Birmingham. A newspaper has published the photograph. During her career the actor has disclosed very little about her private life.

Which of the following best describes whether the actor can bring a claim in the High Court for misuse of private information arguing that her Convention rights have been infringed?

A. She cannot do so. She can only bring a claim arguing that her Convention rights have been infringed before the European Court of Human Rights.

B. She cannot do so because freedom of expression is a more important right than the right to a private life.

C. She cannot do so because English law does not recognise a tort of privacy.

D. She can do so because the right to a private life is entitled to greater protection than freedom of expression.

E. She can do so because the court in making its decision must protect her right to a private life.

A

Option E is correct.

The photograph, relating to a health condition, clearly engages Article 8, the right to a private life. In publishing the photograph the newspaper will be exercising its Article 10 right of freedom of expression. Neither right has precedence over the other, hence options B and D are wrong.

Instead, the court will balance the actor’s Article 8 right with the newspaper’s Article 10 right. In this instance, following the House of Lords’ judgment in Campbell, the balance is likely to fall in favour of the actor’s Article 8 right as the newspaper does not seem to have a legitimate reason for disclosing her medical condition.

Option A is wrong as the actor will be able to bring a claim in an English court under the horizontal effect principle.

Option C is wrong. Following the Supreme Court judgment in in PJS v News Group Newspapers, it is arguable that English law does recognize a tort of privacy. In any event, under the horizontal effect principle the courts will give effect to Convention rights through developing existing causes of action compatibly with the ECHR.

38
Q

A woman was convicted of murdering a child many years ago when she herself was a child. At the time an injunction was granted preserving her anonymity. The woman is due to come up for parole and a media outlet has applied for the injunction to be lifted so that it can give details of the woman’s identity. The woman suffers from severe mental health issues and has attempted suicide on a number of occasions. Medical evidence is submitted that if the injunction is lifted the woman is likely to attempt suicide again. There is no evidence that, should the woman’s identity be released, she would be harmed by others.

Is the media outlet likely to be successful in its application to lift the injunction preserving the woman’s anonymity?

A. Yes, because the woman is now an adult and her Article 8 rights are unlikely to continue to take priority over the media outlet’s Article 10 rights.

B. Yes, because there is no evidence that the woman would be harmed by others if her identity was released and so her rights under Article 2 and Article 3 are not engaged.

C. Yes, because, as the woman is coming up for parole and may be released, it is in the public interest that her identity is known.

D. No, because there is a real risk of serious harm or death to the woman if her identity is released and her rights under Article 2 will take priority over the media outlet’s Article 10 rights.

E. No, because there is no evidence that disclosure of the woman’s identity is necessary to protect the public.

A

The correct answer is D.

There is a risk of serious harm or death to the woman if her identity is disclosed. Her absolute rights under Article 2 will take priority over the media outlet’s Article 10 rights. It makes no difference that the risk of harm comes from the woman herself (D v Persons Unknown; F v Persons Unknown).

A is wrong because the conflict is not between the woman’s Article 8 rights and the media’s Article 10 rights, but between the media outlet’s Article 10 rights and the woman’s rights under Article 2 (and possibly Article 3).

B is wrong because the woman’s rights under Articles 2 and 3 can be engaged when the risk of harm comes from herself (D v Persons Unknown; F v Persons Unknown).

C is wrong as, even if it was in the public interest for the woman’s identity to be known, her absolute rights under Article 2 would still take priority.

E is wrong because the media outlet would not need to produce evidence that disclosure of the woman’s identity is necessary to protect the public to be successful. However, they will be unsuccessful as the woman’s absolute rights would take priority.

38
Q

A well-known actor is photographed without his knowledge while out walking in the park with his new partner. The actor has always co-operated by giving the media interviews about his acting work but has made it clear that his private life is off limits. The photograph is published on the website of a national newspaper under the section on ‘Latest Celebrity Gossip’ with the caption ‘his love-life appears to be blooming along with the flowers this Spring!’. The actor asks for the photograph to be removed but this is refused, so he brings a claim for misuse of private information.

Which of the following best describes the legal position in relation to the actor’s claim for misuse of private information?

A. As the photograph was taken in a public place the actor will have no reasonable expectation of privacy and so his Article 8 rights will not be engaged.

B. The actor may have a reasonable expectation of privacy and, as the photograph does not contribute to a debate of general interest, his Article 8 rights are likely to be given priority over the Article 10 rights of the national newspaper.

C. The actor may have a reasonable expectation of privacy but, as publication of the photograph would not be considered highly offensive to a reasonable person, the Article 10 rights of the national newspaper will be given priority over his Article 8 rights.

D. As the actor and his partner are both adults and the photograph is genuine the actor is likely to fail in his claim for misuse of private information.

E. As the actor did not consent to the photograph being taken the national newspaper will be found liable for misuse of private information.

A

The correct answer is B.

The actor may be able to show he has a reasonable expectation of privacy as he has always made it clear that his private life is off limits. The photograph does not contribute to a debate of general interest. That factor, combined with the actor’s lack of knowledge and consent mean that the balance is likely to come down in favour of the actor’s Article 8 rights.

A is wrong as the fact that the photograph was taken in a public place does not necessarily mean that the actor will have no reasonable expectation of privacy in relation to it.

C is wrong as, although it may be relevant whether publication of the photograph would be considered highly offensive to a reasonable person, this is not the only relevant factor and may not be determinative.

D is wrong as, although the fact that the actor and his partner are adults and the photograph is genuine are likely to be relevant factors, they are not the only relevant factors.

E is wrong as lack of consent is a relevant factor but is not the only relevant factor and is unlikely to be decisive on its own.

39
Q

A well-known celebrity runs a healthy eating blog. A newspaper publishes an article stating that the celebrity has regular unhealthy take-aways delivered from a local restaurant. The celebrity brings an action for misuse of private information.

Which of the following best describes how the court will decide the case?

A. The court will decide the case without reference to the Article 8 rights of the celebrity as the newspaper is not a public authority.

B. As the case involves a conflict between the Article 8 rights of the celebrity and the Article 10 rights of the newspaper the court will decide whether any interference is proportionate by applying the factors set out in the Bank Mellat case.

C. As the case involves a conflict between the Article 8 rights of the celebrity and the Article 10 rights of the newspaper the court will look at the importance of the rights being claimed in the individual case and the justifications for interfering with or restricting those rights and will apply the proportionality test to each.

D. As the case involves a conflict between the Article 8 rights of the celebrity and the Article 10 rights of the newspaper the court will look at the importance of the rights being claimed in the individual case and the effect of interfering with or restricting those rights and will apply the proportionality test to each.

E. The court must have particular regard to the importance of the convention right to freedom of expression so that the Article 10 rights of the newspaper will be given more weight when assessing the balance between Article 8 and Article 10.

A

Option C is correct as it correctly sets out the approach in the case of Campbell v MGN to be adopted when there is a conflict between two qualified rights.

A is wrong. Although the celebrity could not bring a case against the newspaper for breach of their Article 8 rights directly, the court is a public authority and so must give effect to those rights when deciding the action for misuse of private information.

B is wrong as although the factors in Bank Mellat are relevant when someone brings a case against the state for breach of their convention rights, they are not relevant in cases between private individuals where their rights are in conflict.

D is wrong as it does not accurately reflect the approach the court will adopt as set out in the case of Campbell v MGN.

E is wrong as, although the court must have particular regard to the importance of the convention right to freedom of expression under s12(4) Human Rights Act, this does not mean that Article 10 will be given more weight (Campbell v MGN).

40
Q

A man convicted of murder committed when he was an adult is released after serving his sentence. He changes his name and goes to live in a part of the country where he will not be recognised, as he does not want the community in which he is living to know about his past. Some newspapers have found out where he lives and want to publish the details.
Will the man be able to obtain an injunction stopping the newspapers from disclosing his identity and where he lives?

A. Yes, because publication of the information will violate his right to life and right to privacy.

B. Yes, because although publication of his details will not violate his right to life, it will be a disproportionate interference of his right to privacy.

C. Yes, because although publication of his details will not violate his right to privacy, it will be an interference with his absolute right to life.

D. No, because as he committed the murder as an adult, he has forfeited his right to privacy and there is no interference with his right to life.

E. No, because publication of the information will not violate his right to life nor be a disproportionate interference with his right to privacy.

A

Option E is correct. Based on the Mary Bell case, it seems unlikely that the risk of harm to the man will reach the threshold to engage Article 2 (the right to life). As regards privacy (Article 8), freedom of expression of the press is highly important and can only be interfered with in exceptional circumstances, such as existed in the Mary Bell case. No such circumstances seem to exist here.

Options A, B and C are therefore wrong because they suggest either that there has been an interference with both Articles 2 and 8 (option A) or that one of them (options B and C) have been interfered with. As explained above it is unlikely that either has been interfered with.

Option D is wrong as individuals do not forfeit their rights because of criminal conduct.

41
Q

Which of the following statements is correct?

A. Judicial review is a system that allows an individual to appeal from a decision of a public body to the High Court.

B. In a claim for judicial review the court is not concerned with the merits of a decision.

C. It is possible for a claimant to bring a judicial review action to challenge the validity of an Act of Parliament.

D. Where a public body has taken an irrational decision, the court can, on a claim for judicial review, substitute its own (rational) decision.

E. A court will not overturn the decision of a government minister on judicial review as this would be in breach of the doctrine of the separation of powers.

A

the correct answer is B.

In a claim for judicial review the courts are concerned with the legality, rather than the merits, of a decision.

Judicial review is not the same as an appeal – so option A is wrong. Option C is wrong as a claimant cannot challenge an Act of Parliament by way of judicial review due to the doctrine of parliamentary sovereignty. (The claimant could seek a declaration of incompatibility pursuant to s 4 Human Rights Act 1998, but such a declaration does not affect the validity of the Act.)

Option D is wrong as the court cannot substitute its own decision on judicial review, but must refer the matter back to the relevant public authority to make the decision again.

Option E is wrong as the court does have the power to overturn the decision of a government minister on an application for judicial review. This ensures that the government minister stays within the powers given by the legislature and is an example of the separation of powers at work.

42
Q

Assume that under statutory powers a local authority is empowered to purchase land compulsorily where it is satisfied that the land is needed for the development of the area. It orders the compulsory purchase of land belonging to Shaw & Pearce Ltd (S&P) on the grounds that it is suitable for development of the area and to prevent the continuation of its current use by a payday lender. S&P wants to challenge the local authority’s decision by way of judicial review, invoking illegality as a ground.

Which ONE of the following statements is WRONG?

A. S&P will be unable to challenge the decision as one of the purposes pursued by the local authority (the development of the area) was an authorised purpose.

B. Unless the authorised purpose was the primary purpose being pursued by the local authority, the decision will be illegal.

C. The decision will be illegal because the local authority has taken an irrelevant consideration into account, the current use of the land by a payday lender.

D. If the unauthorised purpose (preventing the land’s continued use by a payday lender) materially influenced the local authority’s decision, the decision will be illegal as the local authority was pursuing an improper purpose.

E. If preventing the land’s continued use by a payday lender materially influenced the local authority’s decision, it can be challenged both on the ground of improper purpose and for taking an irrelevant consideration into account.

A

Option A is the one that should be selected. S&P will be able to challenge the decision of the local authority even though one of the purposes the authority was seeking to achieve was an authorised purpose.

All the other options are correct. If, as seems likely, the unauthorised purpose materially influenced the decision to compulsorily purchase S&P’s land, the decision will be illegal (R v ILEA, ex p Westminster City Council).

The courts have also used the primary purpose test where decision makers have pursued two purposes, one authorised and the other unauthorised (Westminster Corporation v LNWR). The current use of the land by a payday lender can also be analysed as an irrelevant consideration (Padfield).

43
Q

A local authority has been given power to issue grants to people to help them insulate their homes. The local authority wishes to adopt a policy stating that priority for such grants will be given to elderly applicants.

Which of the following statements best represents the legal position?

A. The local authority must not form such a policy since it must be able to exercise its power freely in every case.

B. The local authority is free to form such a policy since it has a great many decisions to make and cannot always consider every case in full.

C. The local authority is free to form this policy as long as this does not prevent it considering each case on its own merits.

D. The local authority is free to form this policy if the Act granting it decision making power states that it may do so.

E. The local authority must not form such a policy as to do so would be acting outside of its legal authority.

A

The correct answer is C.

Decision makers, such as the local authority, may formulate policies provided that they are consistent with the statute granting the power and applied in such a way that the decision maker can consider each case on its merits (British Oxygen v Minister of Technology).

This principle is of general application and so the relevant statute does not need to provide the power to do so. B provides a fair explanation of the reasons behind the legal rule but is not the best representation of the legal position.

44
Q

A statutory tribunal has power to fix rent for unfurnished lettings. The tribunal fixes the rent in one case where the landlord has provided a full set of furniture on the basis that the flat is unfurnished.

Which one of the following statements is CORRECT?

A. The statutory tribunal has made an error of law. As this error is jurisdictional the decision is unlawful.

B. The statutory tribunal has made an error of law. As this error is not jurisdictional the decision is lawful.

C. The statutory tribunal has made an error of fact. As this error is not jurisdictional the decision is lawful.

D. The statutory tribunal has made an error of fact. As the error is not a material one leading to unfairness the decision is lawful.

E. The statutory tribunal has made an error of fact. As this error is jurisdictional the decision is unlawful.

A

The answer is E.

The statutory tribunal has not misinterpreted the law so A and B are wrong.

The statutory tribunal has made a mistake of fact – it has concluded that the flat is unfurnished. The statutory tribunal has no power to fix the rent if it comes within the definition of ‘unfurnished lettings’ which would not include a property where there was a full set of furniture.

This means that the statutory tribunal has made a jurisdictional error of fact and the decision will be unlawful (ex p. Khawaja).

The decision may also be a material one leading to unfairness, provided that the four criteria in E v Secretary of State for the Home Department are met. This is unknown, so D is wrong.

45
Q

An Act of Parliament states that the Secretary of State for the Environment, Food and Rural Affairs may award grants for the purpose of establishing wind farms provided that the applicant can show that the land has not produced a profit for the previous two accounting years. A landowner applies for a grant. His land did produce a profit last year but he can show that, due to soil erosion, it will not produce a profit in the future. He is turned down by a civil servant in the Department of the Environment, Food and Rural Affairs, as he does not meet the criteria.

Is the decision of the civil servant lawful?

A. No, the decision is unlawful as the civil servant has fettered his discretion by applying the policy too strictly.

B. No, the decision is unlawful as the civil servant has failed to take into account relevant considerations.

C. No, the decision is unlawful as the Secretary of State cannot further delegate decision-making powers which have been delegated to him in the Act.

D. Yes, the decision is lawful due to the Carltona principle.

E. Yes, the decision is lawful due to s 101 Local Government Act 1972.

A

Option D is correct. The civil servant’s decision comes within the exception to the rule against delegation allowing civil servants to take decisions on behalf of the relevant minister. This exception is known as the Carltona principle (from the case of Carltona Ltd v Commissioners of Works).

C is therefore incorrect as there are exceptions to the rule against delegation.

The decision was made by a civil servant rather than an officer of a local authority so s 101 LGA will not apply and option E is incorrect.

A is incorrect as the criteria are contained in a primary Act of Parliament, so they must be applied. This is not a situation where the Secretary of State has adopted a policy.

Similarly, B is incorrect as the civil servant is applying the statutory criteria and has not ignored relevant considerations.

46
Q

An Act of Parliament (‘the Act’) gives to the Secretary of State for the Environment, Food and Rural Affairs (‘the SoS’) certain powers for the purpose of promoting the use of ‘renewable’ energy in preference to fossil fuels. Those powers include the power to require state schools to teach students about the danger of global warming. The SoS uses these powers to issue directions requiring all state schools to teach students about the danger of global warming. The head of the local state primary school is very unhappy about the directions that the SoS has issued. These directions not only detail the dangers of global warming but set out what they state are ‘mandatory measures’ to be taken by all schools immediately in order to fulfil the purposes of the Act. These include the installation of at least four solar panels per school. The school cannot afford the cost of the solar panels out of the school’s budget.

Has the SoS acted ‘ultra vires’ ie beyond her powers?

A. Yes, as the Act does not specifically give her the power to impose this requirement.

B. Yes, as she applied the legislation in too strict a manner.

C. No, as the Act does not specifically prohibit her from imposing this requirement.

D. No, because she considers the solar panel requirement to be essential to the overall purposes of the Act.

E. No, because a reasonable person would consider her actions to be essential to the overall purposes of the Act.

A

Option A is correct.

The SoS has the power to order schools to take steps to teach about the dangers of global warming, but there is nothing in the Act that gives her the power to order the installation of solar panels so her decision is ultra vires (ex p McCarthy and Stone) and C is wrong.

The SoS can probably do things that are ancillary to the power that she does have, but ordering the installation of solar panels goes well beyond that and it is irrelevant that she (or others) consider such actions essential for the purposes of the Act, so D and E are wrong.

B is wrong as the legislation must be complied with strictly or the SoS will be acting unlawfully.

47
Q

Assume that officials of the Department of the Environment, Food and Rural Affairs (‘the Department’) have statutory power to ban movements of cattle which they suspect to be suffering from a contagious disease. An official from the Department orders a farmer not to move any animals to market. The official tells the farmer that the order to ban the movement of his cattle has been made on the instructions of the National Farmers Union (‘the Union’) who have information that the farmer’s cattle may have a contagious disease as they have not been vaccinated, contrary to the Union’s policy.

Which of the following heads of illegality would it be most appropriate for the farmer to use to challenge the official’s decision?

A. Fettering discretion

B. The rule against delegation

C. Error of law

D. Taking account of irrelevant considerations

E. Acting without legal authority

A

Option A is correct.

Fettering discretion is the most appropriate head of illegality for the farmer to use. The official has allowed herself to be dictated to by the National Farmers Union.

Option B is wrong. Whilst the official may have allowed herself to be dictated to, this will not amount to unlawful delegation, as the official herself made the decision. In unlawful delegation, the delegate will make the decision themselves.

Option C is wrong. There does not appear to be an error of law.

Option D is not the best answer. Arguably, the information from the Union may be relevant, but, even if it is not, this would not be the most appropriate ground to rely on.

Option E is wrong. The official does have power to ban the movement of cattle.

48
Q

An Act of Parliament gives to the Secretary of State for the Environment, Food and Rural Affairs (‘the SoS’) certain powers for the purpose of promoting the use of ‘renewable’ energy in preference to fossil fuels. Those powers include the power to make compulsory purchase orders (‘CPOs’) of ‘appropriately located land’ to build wind farms. The SoS has issued a CPO relating to the housing stock on an old local authority housing estate, right in the middle of a busy town and is therefore not an appropriate place to site a wind farm.

Which of the following statements best expresses what must be shown to establish that the decision is unlawful on the basis of irrationality?

A. That the SoS has acted disproportionately in this situation.

B. That the SoS has acted unreasonably in this situation.

C. That the SoS has acted more unreasonably than the average decision maker would in this situation.

D. That the SoS has acted so unreasonably that no reasonable decision-maker would ever have made the decision.

E. That the SoS has acted in a way that a sensible person would not.

A

D is correct.

It is the most accurate summary of the test for irrationality set out in the landmark Wednesbury case.

Options B and C do not reflect the high level of unreasonableness that has to be proved.
Disproportionality is relevant when considering human rights, not irrationality, so A is wrong.

Lord Diplock, when giving his definition of irrationality in the CCSU case did mention the ‘sensible person’, but E does not accurately reflect the test for irrationality set out in that case.

48
Q

Under an Act of Parliament, local authorities may fine owners of dogs where it is proved that they have left their dog mess (faeces) in the park and have not picked it up and placed it in the bins provided for this purpose. A District Council has discovered sheep droppings in the local park. It traces these droppings back to a sheep owned by a local resident. The local resident has been letting his sheep graze in the park. An officer from the District Council serves the local resident with notice that he is being fined under the Act.

Which of the following heads of illegality would it be most appropriate for the local resident to use to challenge the decision?

A. Fettering discretion

B. Unlawful delegation

C. Taking into account irrelevant considerations

D. Pursuing an improper purpose

E. Acting without legal authority

A

Option E is correct.

The District Council is acting without legal authority. The Act only gives it power to fine dog owners where they have left dog mess in the park. It does not say that they can fine sheep owners. To fine the local resident would mean that the District Council is acting beyond its legal powers and its decision is accordingly unlawful.

Option A is wrong. The officer is applying the Act rather than fettering their discretion through the application of any policy.

Option B is not the best answer. It is not known whether the District Council passed a formal resolution that the officer in question could take the decision. If they had not, the decision would not come under the exception in s101 Local Government Act 1972 so it could be challenged on this ground.

However, given that this is an unknown, and it is clear that the District Council has no power to take the decision, unlawful delegation is not the most appropriate head of illegality to use.

Options C and D are not the best answers. Whilst it might be arguable that the District Council is pursuing an improper purpose or taking into account an irrelevant consideration in using its powers to deal with sheep droppings rather than dog mess, acting without legal authority is the most appropriate ground. The reason why a decision-maker is exercising its powers chiefly becomes relevant once it has been established that the power exists.

49
Q

Assume that a statute (fictitious) gives local authorities the power to grant licences to cinemas in their area ‘on such conditions as they think fit’. Exercising this power, a local authority grants a company a licence on condition that no film should be shown at its cinemas unless approval is first obtained from the local Churches Committee. The company objects to this condition.

Which of the following best describes whether the company can successfully seek judicial review of this licence condition?

A. The decision is lawful as the licensing authority had effectively delegated its decision-making power to another body, the local Churches Committee, and the delegation cannot be challenged.

B. The decision is lawful because the delegation was made to a local committee and the statutory exception relating to local bodies applies.

C. There is illegality as the delegation to a religious body (the local Churches Committee) was in breach of the rule against delegation.

D. There is illegality as the licensing authority had delegated its decision-making power to another body which was not a local authority committee or officer.

E. There is illegality as the licensing authority, by delegating its decision-making power to another body, was pursuing an unreasonable purpose.

A

Option D is correct.

The facts are similar to that of Vine v National Dock Labour Board. The licensing authority has delegated its powers in breach of the rule against delegation and the local authority committee exemption does not apply because the decision was delegated to a local churches committee.

Option D is a better answer than option C as the reason the rule against delegation was breached had nothing to do with the nature of the Churches Committee. Option D is also a better answer than option E; on the facts the licensing authority’s purpose is not known, but in any event even if its purpose had been legitimate it would have breached the rule against delegation.

Option A is wrong, as there is a clear breach of the rule against delegation. Option B is wrong as the statutory exception it refers to applies only to committees, sub-committees and officers of local authorities, not to local bodies in general.

49
Q

Parliament has enacted an Act of Parliament to promote the sale of fairly-traded goods (the Act’). The Act establishes an Agency and gives it power to grant government-backed fair trade certification to importers and producers of fairly-traded goods. Products can only be labelled as fairly-traded if the Agency has granted the appropriate certification. The Agency has issued guidelines that it will normally grant fair trade certification to importers and producers who had previously held fair trade certification for three years under a previous scheme (‘the old scheme’). The Agency has recently rejected an application for certification by a company that imports wine from South Africa, because it had only held fair trade certification under the old scheme for two years. The company had pointed out in its application that the vineyards in South Africa from which it imports wine were only purchased by a workers’ co-operative two years ago.

Which of the following statements best summarises the company’s strongest argument for judicial review?

A. The company should argue that the Agency’s decision is illegal as it did not have the power to adopt guidelines as to when it would normally grant fair trade certification.

B. The company should argue that the Agency’s decision is illegal as the guidelines it adopted are unreasonable.

C. The company should argue that the Agency’s decision is illegal as it constitutes an illegal fettering of its discretion.

D. The company should argue that the Agency’s decision is illegal as it is pursuing an improper purpose.

E. The company should argue that the Agency’s decision is illegal as it has taken into account irrelevant considerations.

A

Option C is correct.

The Agency must use its statutory powers in a manner which is consistent with the aims of the Act and in doing so may formulate a policy to follow in the generality of cases, so A is wrong.

The policy appears to be consistent with the Act: the Agency could argue that the policy provides an appropriate means of ensuring that it grants certification to suitable applicants. Thus it has not acted for an improper purpose and D is wrong.
Equally, the policy does not appear to be unreasonable but, even if it was, it would need to be ‘so unreasonable that no reasonable decision maker’ would have made it so B is wrong.

However, while the Agency may adopt a policy, it should be prepared to listen to anyone who has something new to say (British Oxygen). In this instance the company can argue that the Agency should have taken into account the fact that the vineyards were only purchased two years ago by a workers’ co-operative. Therefore, the Agency’s approach constitutes an illegal fettering of its discretion.

In ignoring this point, the Agency has also failed to take into account relevant considerations (Roberts v Hopwood), but there is no evidence that it has taken into account irrelevant ones, so E is wrong.

50
Q

An Act was passed to lessen the frequency of wildfires in the countryside which are started by discarded glass bottles. The Act gives power to the Environment Minister “to ban goods in glass bottles from being sold in village shops and pubs”. The Minister is a passionate environmentalist and is determined to reduce the amount of litter left in the countryside. He bans the sale of all bottled goods (whether contained in glass or plastic bottles) in village shops and pubs. A client, who is a village shopkeeper, is most concerned about the ban since much of her stock consists of goods sold in glass or plastic bottles. The client wishes to contest the decision by way of Judicial Review.

Which of the following combinations best identifies the most appropriate grounds of challenge?

A. Decision made without lawful authority, with an error of law and error of fact whilst taking account of irrelevant considerations and based on an unauthorised purpose.

B. Decision based on an unauthorised purpose, taking account of irrelevant considerations and using wrongful delegation.

C. Decision made with fettering of discretion, error of fact whilst relevant considerations were ignored and was irrational.

D. Decision made with unauthorised purpose, dual purpose whilst taking account of irrelevant considerations.

E. Decision made with fettering of discretion under the dictation of another, based on an error of fact, with a dual purpose whilst taking account of irrelevant considerations.

A

Option A best describes the suite of grounds that are most appropriate for the client:

The Environment Minister used the power to ban glass bottled goods to ban all bottles whether made of glass or plastic. He had no lawful authority to ban plastic bottles (McCarthys and Stone v Richmond).

He appears to have misunderstood the nature of the power granted i.e. that only glass bottles were to be banned and this indicates an error of law (Anisminic).

His ban on plastic bottles throws up a jurisdictional error of fact (Khawaja), since the significant aspect of the bottles is that they are glass and capable of setting a wildfire.

As a passionate environmentalist, he has taken irrelevant considerations into account (Roberts) and has used the power for an unauthorised purpose (Congreve).

Option B is wrong because the decision was made by the Environment Minister who had been given the power by the Act. Therefore, there was no wrongful delegation.

Option C is wrong because there was no fettering of discretion. The Minister’s passionate advocacy of environmental issues is not enough to trigger this ground. It requires a policy to have been put into place which is then applied rigidly. No such policy had been formulated here.

Option D is wrong as the Minister does not seem to be using the power in order to prevent wildfires so dual purpose does not appear to relevant. This answer excludes error of fact and law which are highly appropriate and significant grounds.

Option E is wrong because no-one else told the Minister what to decide, which is the requirement of fettering discretion under the dictation of another. Also, as above, the Minister does not seem to be using the power in order to prevent wildfires so arguments about dual purpose do not appear particularly relevant.

51
Q

A homeowner applies to the local authority under a statutory scheme for a grant to instal solar panels on the roof of his home. The local authority has just informed him by email that his application has been turned down. No reasons were given for this in the email.

Which of the following statements best represents the legal position as to whether the local authority needs to give reasons?

A. There is no duty on the local authority to give reasons for its decisions when exercising a discretionary power such as giving out grants.

B. Local authorities are always under a duty to give reasons for their decisions.

C. There is no general duty on the local authority to give reasons, but the local authority may be under a duty to give reasons if the decision looks clearly wrong.

D. The local authority is only under a duty to give reasons for its decisions if a statute requires it to do so.

E. The local authority is under a duty to give reasons as the decision will result in financial loss to the homeowner.

A

C is correct.

There is no general duty for a decision maker (including a local authority) to give reasons so B is wrong. Such a duty may be imposed by statute but, even if the statute does not require reasons to be given, the court is likely to imply such a duty where the decision looks wrong or ‘cries out for an explanation’ (R v Civil Service Appeal Board ex parte Cunningham), so D is wrong.

Financial loss alone will not automatically result in there being a duty to give reasons, although may do so if the impact on a person’s rights is serious, so E is wrong.

Whether or not there is a duty to give reasons does not depend on whether the power being exercised is discretionary so A is wrong.

52
Q

Assume that a statute gives local authorities power to license street trading through Street Trading Committees. The Street Trading Officer for a District Council has asked his local Street Trading Committee (‘the Committee’) to revoke a trader’s licence on the grounds that she regularly obstructs the road. The trader is the Street Trading Officer’s neighbour and they have had a long running dispute. The trader is allowed to argue her case in full before the Committee, but objects to the fact that when the Committee went into a separate room to make its decision, the Street Trading Officer went too. The Committee decided to revoke the trader’s licence.

Will the decision to revoke the trader’s licence be quashed on judicial review?

A. Yes, because there is a real possibility that the Committee’s decision was biased, its decision can be quashed.

B. No, because the trader will need evidence to show that the Street Trading Officer improperly influenced the Committee’s decision.

C. Yes, because the Street Trading Officer has a direct interest in the outcome of the decision.

D. Yes, because there has been a breach of the fair hearing rule.

E. No, because the trader was able to argue her case in full.

A

A is the correct answer.

This is a case of ‘apparent bias’, so the court will apply the test in Porter v Magill.

It is never necessary to show actual bias, so B is wrong.

The Street Trading Committee is the decision-maker and it does not stand to gain from the decision, financially or otherwise, so C is wrong.

The trader has had the chance to argue her case so there is no breach of the ‘fair hearing’ rule, but this does not mean that she will be unable to challenge the decision on the basis of apparent bias, so D and E are wrong.

53
Q

A licence from the Borough Council is required to run a market stall. Two months ago the Borough Council revoked a woman’s licence following allegations that her cake stall fell below the hygiene standards required. The Council did not give her the opportunity to respond to the allegations (which she denies). The woman has incurred significant losses through being unable to trade.

Which of the following best explains what remedies would be available, if the woman made a successful claim for judicial review?

A. Prohibiting order

B. Quashing order

C. Damages

D. Injunction

E. Quashing order and damages

A

B is correct.

Judicial review would quash the revocation and allow the woman to continue trading.

A is wrong as a Prohibiting Order will order a Public body to refrain from illegal action, which is not relevant.

C and E are wrong as damages are only available if the claimant can establish his private law rights have been infringed and are not available purely for the infringement of a public law right.

D is wrong as this is to restrain the body from illegal action, which is not relevant.

54
Q

Following concern over the standard of care provided by dog kennels, Parliament has passed the Dog Welfare Act 2020 (fictitious) requiring all dog kennels charging for their service to be licensed under the Act. The Dog Kennel Licensing Board (DKLB) was set up under the Act to administer the scheme. The client applied for a licence and was refused last week. They wish to bring a judicial review of the decision refusing them the licence.

Which of the following best explains a requirement which must be met in order to bring a judicial review?

A. The client will need to establish the decision maker is a public authority for the purposes of the Human Rights Act 1998.

B. The client has six months to bring the judicial review claim.

C. The client must establish the decision maker is a public body exercising a private or public law function.

D. The client must establish the decision maker is a public body by reference to its source of power or nature of power.

E. The client must issue the claim for judicial review within the next 11 weeks.

A

D is correct answer.

The client can only bring a judicial review against a public body and the tests for this are the Datafin source of power test and nature of power test. The DKLB was given its decision-making powers by the Act and therefore satisfies the Datafin source of power test.

Option A is wrong. The client will only need to establish the DKLB is a HRA public authority if it is able to run breach of a Convention right as a judicial review ground. Nothing on the facts suggests there has been a breach of any Convention right.

Options B and E are wrong. The judicial review claim must be brought without undue delay, promptly and in any event within three months. If the client does not act without undue delay then they are likely to be refused permission by the court to bring a judicial review.

Option C is wrong since the decision maker cannot be a public body if it is exercising a private law function.

55
Q

To challenge a public body in Judicial Review an aggrieved person must show that it is the correct course of action.

Which of the following statements best describes some of what must be shown?

A. The claimant must make a challenge in Judicial Review within 3 months of the offending decision or action.

B. The claimant must have sufficient interest in the decision or action taken in respect of public law.

C. If Parliament has attached an ouster clause to a decision or action taken under a statutory power, the court cannot review the matter.

D. Pressure groups are unable to challenge decisions or actions unless they are directly affected.

E. The claimant may challenge a public body through Judicial Review for breach of contract.

A

B is the best answer because Judicial Review is only available to claimants who can show that a public law decision or action affects them sufficiently.

A is wrong because it is not accurate. The challenge must be brought promptly, without delay and in any event within 3 months. The court also has a discretion to allow late claims if there are good reasons for the delay.

C is wrong because it is not accurate. The court will test the decision or action as to its validity before determining whether their jurisdiction has been ousted by Parliament. However, the court will abide by partial ousters (time limited clauses).

D is wrong because it is not accurate. Pressure groups may challenge decisions or actions that do not affect them directly if they satisfy the court that they are in the best position to do so.

E is wrong because the decisions or actions of a public body can only be challenged in Judicial Review if it is matter of public law. Contract law is private law.

56
Q

A woman wishes to extend her property by creating an extension and a large basement area. The woman applies for planning permission under the planning acts. Planning permission is granted by the planning committee, which is chaired by the woman’s brother. The woman’s neighbour is unhappy about the grant of planning permission. She is busy preparing for her wedding the following year but, two months after the brother grants permission, she applies for judicial review of this decision.

Which of the following courses of action is the court most likely to take in response to the woman’s application for judicial review?

A. The court will quash the decision for breach of the common law right to a fair hearing.

B. The court will quash the decision for breach of a statutory duty to comply with procedural requirements.

C. The court will quash the decision for breach of the common law rule against bias.

D. The court will refuse permission for judicial review as the application has been made too late.

E. The court will refuse permission for judicial review as the neighbour does not have standing to bring a claim.

A

D is the correct answer because the time limit for claims under the planning acts is six weeks from the date of the decision. The court is unlikely to extend this time limit as wedding plans do not constitute a ‘good reason’ for delay.

Options A and B are wrong as, even if the neighbour had brought the claim within time, they are not the correct grounds of challenge – the woman is not complaining of a breach of statutory procedure or that she has not had a fair hearing.

Option C is wrong. Although this would have been the correct ground of challenge, the court is unlikely to grant permission for judicial review.

Option E is wrong as the woman will have sufficient interest to bring a claim. She is personally affected by the decision given the amount of noise and disruption the building work will cause.

57
Q

An Act of Parliament (‘the Act’) gives the Secretary of State for Education (‘the SoS’) the power to close failing schools by giving one term’s notice (‘a termination notice’). Before the SoS can serve a termination notice on a school, the school must be inspected by a team of inspectors who must prepare a report recommending whether the school should be closed. The Act also states that a copy of the report “must be sent to the school by Royal Mail special delivery”. The school then has 21 days to make representations to the SoS.

A team of inspectors inspected a secondary school and recommended its closure. The report was, however, sent to the school by ordinary post and by email. The school did make representations to the SoS who nonetheless served on it a termination notice. The school now wants to challenge the decision of the SoS.

Which of the following statements best represents the legal position regarding the validity of the termination notice?

A. The termination notice is invalid as, by not sending the inspectors’ report by special delivery, there has been a breach of an important procedural safeguard and therefore breach of a mandatory procedural requirement.

B. The termination notice is invalid as the Act expressly provides the inspectors’ report must be sent to the school by special delivery; accordingly, there has been a breach of a mandatory procedural requirement.

C. The termination notice is invalid as Parliament would have intended the failure to comply with the requirement to send the report by special delivery to invalidate the decision.

D. The termination notice is valid as the requirement to send it by special delivery is merely a directory procedural requirement and its breach does not invalidate the notice.

E. The termination notice is valid as the requirement to send it by special delivery imposed an excessively onerous burden on the SoS.

A

D is correct as a procedural requirement is likely to be mandatory if its breach results in the claimant being substantially prejudiced (Bradbury v LB of Enfield). On the facts, the school has not been prejudiced and there has been a significant effort to comply with the requirement, so it is likely to be directory (Coney v Choyce).

Accordingly, option A is wrong. The procedural requirement of sending the notice by special delivery is likely to be directory, not mandatory.

Option B is wrong. Although the wording of the Act is important, Coney v Choyce shows that it is not conclusive, and the courts take other factors into account, in particular the importance of the safeguard and whether substantial prejudice has occurred.

Option C is wrong. It is unlikely that Parliament intended that a technical breach of the requirement would result in the notice’s invalidity where no harm occurred (R v Soneji).

Option E is wrong as a procedural safeguard cannot be ignored simply because it is onerous; the courts will take into account the other factors referred to in this piece of feedback.

58
Q

An Act of Parliament (‘the Act’) provides that an inquiry shall be held before a draft compulsory purchase order is confirmed. The Act contains no provisions requiring notice of the draft order to be given to those affected. A draft compulsory purchase order is made and an inquiry is held. A man is affected but finds out about the draft order just before the inquiry, and is unable to prepare his case properly. After the inquiry the draft order is confirmed.

Can the man successfully challenge the compulsory purchase order?

A. Yes, because although the statute contains no express notice requirements, there is nonetheless an implied statutory requirement to give notice of the draft order.

B. No, because an inquiry has been held in accordance with the statutory procedure.

C. No, because the statute contains no provisions requiring notice to be given.

D. Yes, because the order can be challenged on the basis that the inquiry was biased.

E. Yes, because the common law rule requiring a fair hearing gives the man a ground of challenge.

A

E is correct. The fair hearing rule can apply where a statute is silent as to procedure. It requires that a person affected by a decision should have a fair chance to put their case.

As the statute does not mention notice, there is no statutory requirement to give this, so option A is wrong.

Options B and C are wrong because, although an inquiry was held in accordance with the statutory procedure and there was no statutory requirement to give notice, the man did not have a fair hearing.

There is no indication on the facts that the person holding the inquiry has any improper interest or prejudice against the man, so option D is wrong.

59
Q

A woman has applied for a licence to run a care home three times. Her applications have each been turned down without an oral hearing. The woman applies for judicial review on the basis that she has not received a fair hearing.

Which of the following statements best explains whether the woman has had a fair hearing?

A. The woman has not had a fair hearing as the decision affects her livelihood and so this is a forfeiture case.

B. The woman has had a fair hearing as she is a applying for a licence which she has not held before and so this is an application case.

C. The woman has not had a fair hearing as she cannot put forward her case persuasively without an oral hearing.

D. The woman is not entitled to a fair hearing as she has not held a licence to run care homes before.

E. The woman has not had a fair hearing. This is the third time she has applied for a licence and been turned down so the decision-maker must be biased against her.

A

B is correct. The woman’s case is an application case as she has not held a licence before, so option A is wrong. The facts are similar to those of McInnes v Onslow-Fane.

She is entitled to a fair hearing, so option D is wrong. However, a fair hearing in an application case requires only that she has her case decided honestly and without bias, so C is wrong.

There is no indication on the facts that the body granting the licence has any improper interest or prejudice against the woman so E is wrong.

60
Q

The local authority has made a compulsory purchase order (‘CPO’) in relation to a farmer’s property. The relevant statutory rules provide that an inquiry must be held before a CPO is made, but this was not done. Local authority workers have attended the property and, when the farmer refused them entry, broke down the gate and began to set up digging equipment on the land. The farmer applied for judicial review and was successful.

Which combination of remedies should the farmer seek in the judicial review proceedings?

A. A quashing order and damages.

B. A quashing order and a mandatory order.

C. A quashing order, a mandatory order and damages.

D. A quashing order and a prohibiting order.

E. A prohibiting order and a mandatory order.

A

The correct answer is A.

The farmer will wish to quash the compulsory purchase order and claim damages for trespass. This is possible since, if the CPO is found to be unlawful, the local authority workers will have no legal authority to be on the land and so will be trespassing. The court can award damages on a claim for judicial review where, as here, the damages could have been awarded in a civil claim (s31(4) SCA 1981).

The farmer would not need a mandatory order. A quashing order will allow him to retain his land – he does not require an inquiry to be held. Thus B and C are wrong.

It is too late for a prohibiting order since the decision has been made and acted on. Thus D and E are wrong.

61
Q

In accordance with the relevant statute, a man applies to his local council for planning permission to build a factory on land which he has recently purchased. A member of the council’s planning committee lives in a house, which is built next to the land. The man has just received a letter from the council’s planning committee rejecting his application for planning permission.

Which of the following statements best describes whether the man can challenge this decision through judicial review?

A. No, because the man does not have sufficient standing to challenge the refusal of planning permission.

B. No, because the time limit for challenging the refusal of planning permission has expired.

C. No, because a refusal of planning permission cannot be challenged in judicial review proceedings.

D. Yes, because the planning committee may have been biased when refusing the application for planning permission.

E. Yes, because the planning committee has exceeded its powers in refusing the application for planning permission.

A

D is correct.

The member of the planning committee has a direct interest in the rejection of the application for planning permission, because he may not want a factory built next to his home. The decision can therefore be challenged as a breach of the rule against bias, which is one of the rules of natural justice (Dimes v Grand Junction Canal Proprietors (1852) 10 ER 301).

A is wrong. The man has standing to claim judicial review because he is personally affected by the decision (Senior Courts Act 1981, s 31).

B is wrong. The time limit for seeking judicial review (six weeks in a planning case) has not expired, because the man has just received a letter rejecting his application.

C is wrong, as a refusal of planning permission is a matter of public law which can be challenged by way of judicial review.

E is wrong. The planning committee has not exceeded its powers, because it has the statutory power to consider planning applications.

62
Q

Parliament has enacted an Act of Parliament to promote the sale of fairly-traded goods (the Act’). The Act establishes an Agency and gives it power to grant government-backed fair trade certification to importers and producers of fairly-traded goods. Products can only be labelled as fairly-traded if the Agency has granted the appropriate certification. The Agency has issued guidelines that it will normally grant fair trade certification to importers and producers who had previously held fair trade certification for three years under a previous scheme (‘the old scheme’). The Agency has recently refused a company importing coconuts from India fair trade certification even though it had been certified under the old scheme. In giving reasons for its decision, the Agency stated that it had received evidence that the coconuts were no longer fairly-traded and the company had failed to provide sufficient evidence to refute this allegation. The company intends to bring a claim for judicial review of the Agency’s decision.

Is the company’s claim likely to succeed?

A. Yes, because the company has a legitimate expectation that it will be granted fair trade certification and refusing certification is so unfair that it amounts to an abuse of power.

B. No, because although the company has a legitimate expectation that it will be granted fair trade certification, in this instance the court is likely to be confined to reviewing the decision on irrationality grounds.

C. Yes, because the company has a legitimate expectation that it will be granted fair trade certification and refusing certification in breach of the policy constitutes procedural ultra vires.

D. Yes, because the company has a legitimate expectation that it will be granted fair trade certification and, by refusing certification in breach of the policy, the Agency would appear to be biased against the company.

E. No, because as the policy was not set out in statute the company does not have a legitimate expectation that the Agency should have regard to it.

A

B is correct.

A legitimate expectation arises where a party has been given an expectation that a body will act in a given way. Here, the Agency’s policy has probably resulted in a legitimate expectation on the company’s part that it will be granted fair trade certification.

The company’s legitimate expectation is likely to fall into the category under which the Agency is merely required to have regard to its policy or other representation. It must give its policy appropriate weight before deciding whether to depart from it, and the court is restricted to reviewing the decision on Wednesbury irrationality, grounds.

As there is evidence that the company’s coconuts are no longer fairly traded, its claim is likely to fail.

Option A is wrong. It is based on the category of abuse of power. However, this category of legitimate expectation only arises where the public body concerned has made a specific undertaking, directed at a particular individual or small class of people, that the relevant policy would be continued. This does not seem to be the case here.

Option C is wrong, as procedural ultra vires arises where a decision-maker has failed to follow a statutory procedure rather than a policy.
Option D is wrong as there is no evidence that would lead an impartial observer to conclude that there was a real possibility of bias.

Option E is wrong as the Agency’s policy has probably resulted in a legitimate expectation on the company’s part.

63
Q

Parliament has passed the (fictitious) Contaminated Land Recovery Act (‘the Act’). The Act allows the local authority to carry out a compulsory purchase of land which has been identified by the local authority as contaminated land. The Act also allows the local authority to issue a preliminary stop notice (‘Notice’) in relation to any land which it potentially considers to be affected by contamination. The Notice restricts the use of the land temporarily and only affects the land whilst the local authority is carrying out its investigation regarding the contamination. The solicitor is acting for a client who has just received a Notice which has been issued regarding his land. The client was not aware of the local authority’s intention to serve the Notice and had no opportunity to make any representations in that regard. The client considers he has not had a fair hearing and wants to bring a judicial review claim on that basis.

Can the client make a claim for judicial review on the basis of a right to a fair hearing, in relation to the issuing of the Notice?

A. Yes, because the client is a forfeiture claimant and therefore is always entitled to a right to a fair hearing.

B. Yes, because the client has sufficient interest and therefore has standing to make the claim.

C. No, because the matter is a preliminary matter and therefore the right to a fair hearing does not apply.

D. No, because the client is a mere applicant claimant and therefore not entitled to a fair hearing.

E. No, because the statute would normally include a full ouster provision which would mean the client is unable to bring such a claim.

A

C is correct.

The right to a fair hearing is a procedural ground for judicial review. The Notice is just a temporary notice. The final decision to compulsorily purchase the land had not yet been made. The decision is merely preliminary and therefore the client is not entitled to a right to a fair hearing at this stage (Lewis v Heffer [1978] 1 WLR 1061).

Option A is not the best answer. Whilst the client may be losing the use of its land, the matter is still preliminary in nature and therefore the client does not have a right to a fair hearing at this stage.

Option B is wrong. The client may have sufficient interest to have the standing to bring a claim in judicial review, but this does not automatically mean that the client will have a legitimate challenge.

Option D is wrong. The client is not applying for a licence, membership or office and is therefore not an applicant claimant.

Option E is wrong. The facts make no reference to an ouster clause in the Act and in any event, full ouster clauses do not mean that the decisions is immune from challenge (Anisminic v Foreign Compensation Commission [1969] 2 AC 147).

64
Q

A statute provides that, if a local authority is satisfied that a person is homeless, it must provide suitable accommodation. X says that the accommodation he has been given is not suitable.

Indicate whether the following statement is true or false.

On judicial review a court would receive evidence of the suitability of the accommodation for X and, if it is not suitable, direct what accommodation should be provided.

A

The statement is false.

What is suitable is clearly a matter of discretion for the local authority. A court would be most unlikely to receive evidence on such a question. However it could quash a decision which it found was irrational. A court would not direct what accommodation should be provided as this would effectively substitute its own decision for that of the decision-maker, in breach of the separation of powers.

65
Q

It is a criticism of the common law theory that this theory justifies unelected judges making law.

Is this statement true or false

A

The statement is true.

The common law theory states that the grounds for judicial review are judge-made and can be developed by the courts in line with the principles of good and fair administration.

Critics argue that this gives too much power to make law to the unelected judges contrary to the principle of the separation of powers.

66
Q

Indicate whether the following statement is true or false.

The judiciary’s traditional justification for judicial review is that it is important that Parliament should not be able to pass any primary statute it wants, unchallenged.

A

The statement is false.

In fact, the judiciary’s traditional justification for judicial review is quite the opposite, namely that it is upholding Parliamentary Supremacy.

67
Q

When a court judicially reviews a decision made by a Government Minister using their powers under the Royal Prerogative, the court is helping to uphold the will of Parliament by ensuring that the Minister does not exceed or abuse those powers granted to them by Parliament.

Is this statement true or false?

A

The statement is false.

Powers exercised by Government Ministers pursuant to the Royal Prerogative are non-statutory in nature - such powers do not derive from an Act of Parliament.

67
Q

Is the following statement true or false:

In judicial review proceedings, the court is concerned primarily with the manner or procedure by which a decision has been reached. The court will not examine the merits of the decision.

A

The statement is true.

In judicial review proceedings, the court is concerned only with the manner in which a decision was made or the procedure that was followed. Were the court to go on to consider the actual merits of the decision, this would usurp the role of the decision maker in breach of the separation of powers.

68
Q

A client, who comes from Nepal, wishes to challenge a decision by the Home Office to deport him once he has been released from prison after serving a sentence of 12 weeks for a common assault. The Home Secretary has the power to deport a foreign national who has received a custodial sentence of at least 12 months. The decision was taken by a civil servant in the Home Office without a hearing.

Which is the best set of potential grounds on which he could challenge the decision?

A. Relevant considerations ignored, wrongful delegation and error of fact.

B. Error of fact, error of law and breach of natural justice.

C. Wrongful delegation, irrationality and bias.

D. Procedural ultra vires, wrongful delegation and error of fact.

E. Error of law, fettering of discretion and breach of natural justice.

A

B is correct.

The civil servant appears to have made an error either of fact with respect to the length of the client’s sentence or has made a misread the law (or both). In addition, the client would be entitled to a fair hearing which he has not had.

A is wrong since a civil servant is able to exercise the powers of a minister of State so there is no wrongful delegation. Ignoring relevant considerations and error of law are potentially good grounds.

C is wrong since a civil servant is able to exercise the powers of a minister of State so there is no wrongful delegation. There is no indication of bias or irrationality.

D is wrong, because there is no indication of a breach of a statutory procedure and there is no wrongful delegation answer since a civil servant is able to exercise the powers of a minister of State. Error of fact is a potential ground.

E is wrong, because there is no policy under which discretion could be fettered. Error of law and breach of natural justice are potential grounds.

69
Q

In order to address concerns about the behaviour of nightclub door supervisors in controlling patrons, Parliament passed the (fictitious) Regulation of Nightclubs Act 2016 (‘the Act’). The Act creates the Entertainment Conduct Authority (the Authority) to regulate the nightclub industry and to grant licences to individuals. Every door supervisor must hold a licence from the Authority to work in the industry.

The Nightclub Workers’ Federation (NWF) is concerned about the fee that the Authority is asking its members to pay on applying for their licences, believing it has exceeded its statutory powers by charging a fee. The NWF wishes to challenge this on their behalf.

Can the NWF seek judicial review of the licence fee?

A. No, because the Authority is not amenable to judicial review claims, nor does the NWF have sufficient interest to apply for judicial review.

B. No, because although the NWF does have sufficient interest to apply for judicial review, the Authority is not amenable to judicial review claims.

C. No, because although the Authority is amenable to judicial review claims, the NWF is unlikely to have sufficient interest to apply for judicial review.

D. Yes, because the Authority is amenable to judicial review claims, and the NWF is likely to have sufficient interest to apply for judicial review.

E. Yes, because as the NWF has sufficient interest to apply for judicial review, the Authority will be deemed to be amenable to judicial review claims.

A

Option D is correct.

In order to seek judicial review of a decision, there must be a public body carrying out a public function. Here, the Authority is empowered to grant licences, so both elements are satisfied (ex p Datafin). The Authority is therefore amenable to judicial review.

As regards standing, the NWF is a body representing a number of members but, not being able to apply for a licence, is not itself directly affected by the decision.

As such, it will have to show that it has ‘sufficient interest’ to challenge the decision relating to application fees, and the factors from World Development Movement [1995] 1 WLR 386 will assist. The court will consider the need to uphold the rule of law, which here involves what appears to be an arbitrary attempt to impose a fee (Congreve v Home Office).

The importance of the issue, which in this instance is whether or not power is given in the statute to charge a fee and the role of the pressure group, which is to promote its members interests. Although individual members could bring a challenge themselves it is unlikely that its members would themselves have the resources to mount an individual challenge. In addition, the NWF are likely to have substantial expertise. As such, the NWF is likely to be permitted to seek judicial review.

Option A is wrong because it incorrectly states the position regarding both amenability and standing. Although option B correctly states the position regarding standing, it is wrong regarding amenability. Option C is correct regarding amenability, but wrong regarding standing.

Option E is wrong. Although it correctly states that the NWF has standing, it does not therefore follow that the decision-maker is amenable to judicial review claims.

70
Q

Assume that a statute (fictitious) gives power to the Secretary of State to assess claims for property damage arising out of terrorist bomb attacks and to award financial compensation based on the damage suffered. The Act permits a maximum award of £100,000 to be made.

A charity which provides temporary accommodation for the homeless had its premises destroyed in a terrorist bomb attack. It applied to the Secretary of State for compensation of £100,000 to go towards the cost of rebuilding, estimated at £150,000. It was awarded compensation of only £5,000. No reasons were given by the Secretary of State for the size of the award. Before applying for compensation, the charity consulted the Secretary of State’s department and was sent a copy of a departmental circular which states ‘… when awarding compensation, a full award will normally be given to charitable organisations providing care for disadvantaged groups’.

Which of the following best describes the likely result of any claim for judicial review of the amount of the award and the reason for it?

A. The charity’s claim is likely to fail as it is a mere applicant and the Secretary of State’s only duty is to act honestly and without bias. On the facts provided, there is no evidence of dishonesty or bias.

B. The charity’s claim is likely to fail. The Secretary of State is under no duty to give reasons and, in the absence of reasons, the charity cannot prove the Secretary of State has acted illegally or irrationally.

C. The charity’s claim is likely to succeed. By failing to award £100,000 compensation, the Secretary of State has breached a mandatory procedural requirement.

D. The charity’s claim is likely to succeed. The statement in the circular is likely to create a substantive legitimate expectation that its application will be successful and it will be an abuse of power to frustrate that expectation.

E. The charity’s claim is likely to succeed. In the absence of reasons, the decision is likely to be held to be irrational.

A

E is correct.

The charity will most likely be able to bring a claim based on irrationality. While there may also be procedural impropriety, neither options C nor D correctly summarise the position.

The charity is entitled to a fair hearing. Initially it seems that it is a first-time applicant for compensation (McInnes v Onslow-Fane). However, the statement in the circular is likely to have created a substantive legitimate expectation that its application will be successful.

It is then necessary to analyse whether this case falls within the first or third of Lord Woolf’s categories in Coughlan. It is unlikely to fall within the first category as it is not a specific undertaking addressed to an individual or small group. The charity would therefore have to rely on irrationality. As no reasons have been given for reducing the award of compensation from £100,000 to £5,000, the charity has a strong argument that the Wednesbury threshold of irrationality has been reached. Option E is therefore correct and a better answer than option D. Option B is also clearly wrong as the absence of reasons is likely to lead to a finding of irrationality.

Although there is no general requirement for Ministers to give reasons for their decisions, the courts may require this if the decision appears wrong. Here, they may require reasons to enable the charity to ascertain whether the Minister took all relevant circumstances into account in reaching his decision (ex p Cunningham). In this case, the size of the award may appear unjustifiably low, and if so, the decision could be quashed for the failure to give reasons. However, this duty arises from the common law rules of procedural fairness, and do not arise from statute; hence option C is wrong. Option A is wrong as the circular has created a legitimate expectation that the charity will receive a full grant.

71
Q

Assume that a statute (fictitious) gives local authorities the power to order the closure of market stalls in a public market if the trader has repeatedly sold goods which are not of a satisfactory quality. The statute provides that the operator of any stall which is to be the subject of a closure order shall be given seven days’ notice of the order and shall also be given the right to make representations against the closure. Using its power, the local authority has sent a notice to a trader ordering the closure of his stall after seven days. The local authority has stated that due to the poor quality of the goods the trader sells, there is no point in him making representations.

Which of the following best describes whether the trader could seek judicial review of the closure notice?

A. The trader’s claim is likely to fail. Due to the seriousness of the matter, the local authority was entitled to dispense with the requirement to allow the trader to make representations.

B. The trader’s claim is likely to fail. The requirement to allow the trader to make representations is merely a guidance as to good practice and its breach does not render the closure notice invalid.

C. The trader’s claim is likely to fail. The requirement to allow the trader to make representations is merely a directory procedural requirement and its breach does not render the closure notice invalid.

D. The trader’s claim is likely to succeed. The requirement to allow the trader to make representations is a mandatory procedural requirement as Parliament probably intended its breach to invalidate the closure notice.

E. The trader’s claim is likely to succeed. The local authority has acted without legal authority in ordering the closure of the stall without allowing the trader to make representations.

A

D is the correct answer.

The issue is whether the local authority has failed to comply with a mandatory procedural requirement or a directory one. Non-compliance with the former renders a decision invalid on grounds of procedural ultra vires, whereas failure to comply with a directory requirement does not. An important factor that the court will take into account is the wording of the statute itself. According to the facts, the statute provides that local authorities ‘shall’ allow operators of stalls to make representations; this points towards a mandatory obligation to consult. However, the language used is not conclusive.

Case law also shows that where a claimant is substantially prejudiced by non-compliance with an important procedural safeguard, the courts are likely to rule a statutory requirement is mandatory. Another question is whether Parliament would have intended the consequence of non-compliance with the relevant statutory requirement to be the invalidity of the decision. This seems likely, as closure of the stall will deprive the trader of his livelihood. Accordingly, options C and B are wrong for suggesting the requirement is merely directory or guidance as to good practice respectively.

Option A is wrong as, in the absence of statutory authority, the seriousness of the matter does not dispense with the need to observe procedural requirements.

Option E is wrong as the local authority did have the legal authority to order the closure of the stall, provided it followed the correct procedure. Acting without legal authority (one of the headings under illegality) arises when a decision-maker does not have the power at all to take a given decision, no matter how properly it tried to act.

72
Q

An established and successful public limited company (‘PLC’) applied to the Railway Authority to be awarded the franchise to run the local area railway. Its application was turned down last week. PLC has just found out that the chair of the Railway Authority, is married to the chief executive officer and major shareholder of one of PLC’s leading commercial rivals. PLC would like to bring a claim for judicial review.

What advice should be given to PLC?

A. PLC’s claim is likely to succeed as the evidence proves the existence of direct bias.

B. The chair of the Railway Authority does not have a direct interest in the matter, thus there is no presumption of bias and the decision will not automatically breach the rule against bias.

C. A fair-minded and impartial observer would conclude that there was a ‘real possibility’ that the decision-maker had been biased and so the decision will breach the rule against bias.

D. PLC will need to prove that a fair minded and informed observer would naturally conclude that there was a conflict of interest for the decision to breach the rule against bias.

E. The Railway Authority will be able to successfully defend a claim for judicial review if it can prove that its chair was completely impartial and did not influence the decision.

A

Option C is correct.

Due to the closeness of the relationship (husband/wife), this may well be a case of a direct interest, but even if the chair’s interest fell short of that, PLC could still argue that there was sufficient evidence to lead a fair-minded and impartial observer to conclude that there was a ‘real possibility’ that the decision-maker had been biased (Porter v Magill), given that the spouse’s company stood to gain a competitive edge from the decision.

Option A is wrong as there is no need to prove actual bias. Conversely, even if it can be shown that the chair was not biased, this will not provide a successful defence as the court is concerned with the appearance of bias, so E is wrong.

B is wrong as the chair could have a direct interest in the matter eg if he too is a shareholder in the rival company, and that would create a presumption of bias (Dimes v Grand Junction Canal Proprietors).

D sets out the wrong test for indirect or apparent bias.

73
Q

The UK government has power under an Act of Parliament (‘the Act’) to give grants to businesses for the purpose of carrying out research into renewable energy sources. The government has adopted a policy of only considering applications for a grant where an applicant has a record of research into renewable energy and has been in business for at least 10 years. A company has been in business for nine years and has won several prestigious industry awards for its work in renewables research. The government refuses the company’s application for a grant. The company wants to challenge the government’s decision in the High Court.

Which of the following submissions will most likely persuade the High Court that the government’s decision was unlawful?

A. The government’s decision is likely to be unlawful because it is pursuing an improper purpose under the Act.

B. The government may have unlawfully fettered its discretion by the application of its policy in these circumstances.

C. The company has a substantive legitimate expectation that the government will give it a grant which it would be unlawful for the government to frustrate.

D. It was unlawful for the government to adopt any policy about its approach to determining applications for grants.

E. The decision may be a breach of natural justice, because a fair-minded and informed observer would consider that there was a real possibility of bias on the government’s part.

A

B is correct.

While it is lawful for a public body to adopt a policy as to how it will exercise its discretion, it must be ready to listen to someone “with something new to say” and consider departing from its policy in relevant cases (British Oxygen Limited v Minister of Technology [1971] AC 610). For that reason option B is the most likely ground to succeed here.

Option D is wrong in principle as it is lawful for a public body to adopt a policy.

Option A is wrong. While public bodies must not use statutory powers to pursue improper purposes (Congreve v Home Office [1976] QB 629), the government is not obviously doing so here.

Option C is wrong because substantive legitimate expectations arise rarely (see R v North and East Devon Health Authority; ex parte Coughlan [2001] QB 213) and certainly not on these facts.

Whilst Option E sets out the test for bias from Magill v Porter [2002] 2 AC 357, it is wrong because on the facts there is no arguable case of bias here.

74
Q

A planning application to develop a disused chalk pit into a lorry park has been rejected in a letter from the Minister for the Environment who has statutory power to grant or refuse such applications. The Minister refused to grant planning permission on the grounds that it was her policy to seek approval for such plans from the Minister of Housing who may wish to zone the chalk pit for residential development. The Minister of Housing refused his approval on the basis that the land was part of a proposed redevelopment plan for the area.

Can the Minister of the Environment’s decision be challenged in Judicial Review?

A. Yes, although she is entitled to formulate a policy to suit her discretionary power to grant or reject planning permission, she must not fetter her discretion by acting under the dictation of another.

B. Yes, she has introduced irrelevant considerations by taking account of the Minister of Housing’s objection and has exhibited bias in her decision.

C. Yes, she has breached the rules of natural justice by making such an important decision without offering an oral hearing.

D. No, she is a Minister of State who sits in a cabinet of ministers who must take decisions together in order to produce ‘joined up government’.

E. No, the land has already been zoned by the Minister of Housing for a residential development which is more important than a lorry park.

A

A is the correct answer because Ministers of State may formulate a general policy to make it easier for discretionary power to used fairly and equally. She may not, however, fetter her discretion by allowing someone else to dictate how to use that power.

B is wrong because, although the Minister of Housing’s opinion might be irrelevant, there is no indication of bias in the facts.

C is wrong because natural justice is not necessarily breached because the aggrieved person does not receive an oral hearing. The hearing must be fair in all the circumstances so that a decision based on documentation might be sufficient.

D is wrong because the person who is granted the power is the only one who may use it. Although she may take notice of the opinions of others, she must make the decision herself.

E is wrong because whilst it might be correct to say that the land has already been zoned for residential development, the power to grant planning permission had been given to the Minister of the Environment and she should not have fettered her discretion by allowing the Minister of Housing to dictate the decision.

75
Q

A local authority has the statutory power to license ice cream vans to sell ice cream within its boundaries. A woman has applied to the local authority for a licence to operate an ice cream van. The local authority has refused to consider her application. She has lost money as a result of being unable to trade.

What remedy or remedies should the woman apply for?

A. A quashing order

B. A quashing order and damages

C. Damages

D. A mandatory order and damages

E. A mandatory order

A

Option E is correct.

The purpose of quashing orders is to set aside unlawful decisions, but the local authority has refused even to consider the application. A mandatory order will force the local authority to consider the woman’s application lawfully.

Although the woman has lost money due to the refusal to consider the application, she is unlikely to obtain damages. In a claim for judicial review, a claimant can only be awarded damages if they can establish that their private law rights have been infringed. However, damages are not available purely for the infringement of a public law right (ex p Maguire).

Options A and B are therefore wrong because there is not a decision to quash, and additionally option B refers to damages.

Options C and D are wrong because they refer to damages, even though option D does correctly refer to a mandatory order.

76
Q

An environmental group are holding a protest which involves blocking the main ring road around Midchester. Traffic is at a standstill and some of the motorists are becoming frustrated and angry. The police are called and Sergeant Green and Inspector Li attend.

Is the following statement TRUE or FALSE?

As the senior police officer present at the scene Inspector Li will be able to place conditions on the protest if the criteria in s14 Public Order Act 1986 are satisfied.

A

The statement is true. The ‘senior police officer’ in advance of a public assembly or procession is the Chief Constable or Commissioner of Police.

Once the assembly or procession is underway, the ‘senior police officer’ is the most senior officer who is present at the assembly or procession. Here, Inspector Li is of a higher rank than Sergeant Green and is the senior police officer on the scene.

77
Q

The following facts are relevant to questions 2-4:

Kwasi is the leader of the Monarchist Tendency (‘MT’), a group that campaigns for the restoration of an absolute monarchy in the UK. MT plans to hold a protest march in Newlands North, the Prime Minister’s constituency, as the Prime Minister vehemently denounced the MT’s views last week.

The marchers are planning to march past the offices of the Global Party (fictitious), the party which the Prime Minister leads.

Which one of the following statements is correct?

A. Kwasi must give the police reasonable notice of the date, time and route of the proposed procession.

B. Kwasi must give the police at least six clear days’ notice of the date, time and route of the proposed procession.

C. Kwasi is under no obligation to give notice as it is not reasonably practicable to do as the Prime Minister’s denunciation of MT is very recent.

D. Kwasi is under no obligation to give notice as the requirement to give notice interferes with his right to freedom of assembly under Article 11 ECHR.

A

Option B is correct.

Section 11 of the Public Order Act 1986 (‘POA 1986’) requires organisers of public processions to give six clear days’ notice to the police of the event’s location and routes that the procession will follow unless this is not reasonably practicable.

Option A is wrong as s 11 sets out a specific number of days’ notice that organisers must give.

Option C is wrong as it is practicable to give notice, as the march does not appear to be an immediate response to the Prime Minister’s denunciation of the MT; hence it is possible to give the notice.

Although the requirements of the POA 1986 do engage Article 11(1) – freedom of assembly, the requirement to give notice is justifiable. Article 11 is a qualified right and may be interfered with on various grounds, such as national security, public safety or for the prevention of disorder, provided the interference is proportionate. The notice requirement seems proportionate.

78
Q

Assume that the police become aware of MT’s proposed march before it starts. The chief officer of police for the police force in which Newlands is located has received credible intelligence reports that a violent fringe of MT’s membership intends to storm the Global Party’s offices and set it on fire during the march.

Which one of the following is CORRECT?

A. The chief officer of police may give directions banning the march due to the threat to national security.

B. The chief officer of police must give directions banning the march due to the threat to national security.

C. The chief officer of police may give directions ordering the march to take a route that does not go past the Global Party’s offices.

D. The chief officer of police cannot take any action at this stage but must ensure there are enough officers available to deal with any disturbances that arise.

A

C is correct.

Section 12 of the POA 1986 grants the chief officer of police the power to give directions imposing conditions on a public procession if they reasonably believe it will result in serious public disorder; serious damage to property; serious disruption to the life of the community; the noise generated may seriously disrupt the activities of an organisation in the vicinity or cause a significant impact on people nearby either intimidating or harassing them or causing them alarm or distress; or that the organisers have an intimidatory purpose.

The chief of police has received credible reports that some of the MT marchers intend to storm the Global Party Offices and set them on fire. This will provide them with a reasonable belief that the march will result in serious damage to property, so the chief officer of police may give directions to try and prevent this happening and option D is wrong.

Options A and B are wrong as the chief officer of police does not have the power to ban marches. Section 13 of the Public Order Act does empower the chief officer of police to apply to the local council for an order prohibiting public processions in particular circumstances. There is no evidence of any such banning order in this scenario.

79
Q

Assume that the marchers agree to avoid the Global Party’s offices and the march takes place peacefully. When the march ends, about 100 protestors gather in a nearby public park and Kwasi starts a speech in which he abuses the Prime Minister. The protestors start to chant, ‘String him up! String him up!’ When some Global Party officials enter the park on their way home after leaving the offices, the protestors make threatening gestures towards them, shouting ‘we’ll be waiting for you when you come to work tomorrow’.

Sergeant Pearson and PC Ross are watching the events unfold.

Which one of the following statements is CORRECT?

A. PC Ross may order the protestors to leave the park within the next 10 minutes.

B. It is unlikely that the police are able to take any action to end the protest.

C. Sergeant Pearson may order the protestors to leave the park within the next 10 minutes.

D. The police officers at the scene must urgently contact the chief officer of police for the Newlands area to obtain permission to order the protestors to disperse.

A

C is correct.

Section 14 of the POA 1986 empowers the senior police officer at the time a public assembly is taking place to impose conditions if they reasonably believe it may result in serious public disorder; serious damage to property; serious disruption to the life of the community; the noise generated may seriously disrupt the activities of an organisation in the vicinity or cause a significant impact on people nearby either intimidating or harassing them or causing them alarm or distress; or that the organisers have a purpose of intimidation. Once the march has ended, it will have turned into a public assembly as defined by s 16.

In the light of Kwami’s abusive speech and the protestors’ shouts and gestures, it is likely that there are reasonable grounds for believing they have an intimidatory purpose - compelling the Global Party officials not to go to work the next day.

Options B and D are therefore wrong as the senior police officer at the scene does have power to take action. However, option A is wrong as, even if the requisite conditions are satisfied, PC Ross is not the most senior police officer present at the scene. Sergeant Pearson ranks above PC Ross, so PC Ross does not have the authority to impose conditions on the assembly.

80
Q

The local branch of the Save the Badger group is holding a vigil in front of the town hall to call on the government to stop culling badgers. The vigil begins on Sunday evening. A rival group, Halt the Spread, has gathered on the opposite side of the street to oppose them. Tessa leaves her flat above where the Halt the Spread group has gathered and joins them. The Save the Badger group start shouting at them to ‘Go away!’ The Halt the Spread group has been infiltrated by anarchists who break into a shop and steal some eggs. They hand one to Tessa and she and others throw them across the road. One Save the Badger supporter is hit in the eye. The police arrive and arrest Tessa for breach of the peace. Tessa claims that her arrest is unlawful as it breaches her rights under Article 11 of the European Convention on Human Rights

Which one of the following statements is CORRECT?

A. The police cannot lawfully arrest Tessa using their breach of the peace powers as any breach was caused by the Save the Badger group who were shouting at Halt the Spread.

B. The police may have breached Tessa’s rights under Article 11 ECHR if they did not act proportionately.

C. The police are unable to use their breach of the peace powers as a breach of the peace would only have occurred if the owners of the damaged shops had been present at the time.

D. Tessa’s Article 11 ECHR rights will not be breached as they only protect the right to peaceful assembly.

A

D is correct.

Tessa has thrown an egg and so has breached the peace herself, thus option A is wrong.

Although the shop-owners are not present, the violence appears to have resulted in harm as one of the Save the Badger supporters is hit in the eye. A breach of the peace has occurred (R v Howell), so option C is wrong. Article 11 only protects peaceful protests and, as Tessa’s protest is not peaceful, option B is wrong.

81
Q

A district council has lawfully banned a trespassory assembly within four miles of a national monument.

Is the following statement TRUE or FALSE?

As the result of the ban, protestors are banned from protesting on any public highway in the area covered by the ban.

A

The statement is false. In DPP v Jones the House of Lords held that a public highway was a public place that the public might enjoy for any reasonable purpose, provided the activity being carried out did not amount to a public or private nuisance and did not unreasonably obstruct the highway. Accordingly, peaceful, non-obstructive short-term protests on public highways are lawful.

82
Q

The government has introduced legislation into Parliament requiring dogs to be walked on leads at all times when in public places. Nathan has given notice to the police that he intends to hold a march through the centre of Milnerton on Saturday afternoon protesting against this legislation .

The Chief Constable of the police force responsible for Milnerton is concerned about Nathan’s plans as the march is taking place on a Saturday afternoon. It may therefore cause a few short traffic jams and make it harder for people to get to the shops.

Which one of the following statements is WRONG?

A. The Chief Constable has insufficient grounds to apply to the local authority for the march to be banned.

B. The Chief Constable has insufficient grounds to give the marchers directions to avoid the town centre.

C. The Chief Constable can give the marchers directions to avoid the town centre on the basis of the serious disruption to the life of the community the march will cause.

D. The highest ranking police officer present at the time may give directions regarding the route of the march at the time it takes place if sufficient grounds exist.

A

Option C is the option that should be selected, as all the other options are correct. The march will be a public procession as it is taking place on the highway which is a public place under s 16 POA 1986.

The Chief Constable, as the chief officer of police and accordingly the senior police officer in advance of the march, will be able to place conditions on the march if they reasonably believe that it may result in serious public disorder; serious damage to property; serious disruption to the life of the community; the noise generated may seriously disrupt the activities of an organisation in the vicinity or cause a significant impact on people nearby either intimidating or harassing them or causing them alarm or distress; or that the organisers have a purpose of intimidation (s 12 POA 1986).

The Chief Constable is concerned that there will be short traffic jams and shoppers may be inconvenienced, but this is unlikely to amount to ‘serious disruption…’

Option B therefore sets out the position accurately, while option C is wrong as the Chief Constable does not have sufficient grounds.

If the Chief Constable reasonably believes their powers to impose conditions would be insufficient to prevent serious public disorder, they may seek an order from the local authority banning all processions or a class of processions, subject to obtaining the Home Secretary’s consent. This would cover Nathan’s march but there are no grounds for believing there might be serious public disorder. Option A therefore sets out the position accurately.

If on the day of the march any of the conditions set out in s 12 are met, the senior police officer may give directions to the marchers regarding its route. During the march, the highest ranking police officer at the scene will be the senior police officer and so will have the power to give directions. Accordingly, option D is accurate.

83
Q

No Airport Here (‘NAH’) is a group protesting against the building of a new airport near the town of Middlewich. Fifteen members of NAH plan to hold a protest rally outside the local town hall at the time the Planning Committee of Middlewich Council is considering the planning application for the airport. Airport in Middlewich (‘AIM’) is a group which is in favour of building the airport. It has threatened to disrupt NAH’s rally.

Which one of the following statements is CORRECT?

A. The senior police officer cannot impose conditions in advance of NAH’s meeting.

B. The senior police officer can impose conditions on NAH’s meeting if he reasonably believes that it will result in public disorder.

C. The senior police officer can ban NAH’s meeting if he reasonably believes that the conditions he could impose would not prevent serious public disorder.

D. The senior police officer has no power to ban NAH’s meeting.

A

Option D is correct.

The senior police officer can only ban meetings which are trespassory assemblies and must apply to the local authority for an order prohibiting them. In any event, NAH’s meeting will only be attended by 15 people – 20 or more are needed for a trespassory assembly.

This means option C is wrong. Option A is wrong – the police can impose conditions in advance of the meeting or at the time.

Option B is wrong as there needs to be a risk of serious public disorder.

84
Q

A group of about 100 demonstrators have gathered in a square outside a town hall protesting against a local authority’s cuts to library services. The time is nearly 17.00 when many of the local authority’s workers will be leaving the town hall and some of them normally walk through the square on their way home. Most of the demonstrators are chanting slogans such as ‘Save our libraries’ and ‘Down with the council’, but there has been no violence. A police sergeant, supervised by her inspector, are the only police at the scene. The inspector orders the demonstrators to disperse in the next 15 minutes to ensure that the workers can go home without any trouble.

Which of the following best describes whether the inspector’s order is lawful?

A. As the senior police officer present, she can impose a condition requiring the protestors to disperse as the demonstrators’ behaviour will clearly intimidate the local authority’s workers.

B. She has common law powers to order the demonstrators to disperse as she has reasonable grounds for believing that a breach of the peace will occur.

C. Although the inspector is the senior police officer present, she cannot impose conditions on the protest as she does not have reasonable grounds for believing that the demonstrators’ behaviour will intimidate the local authority’s workers.

D. The inspector does not have any common law powers to order the demonstrators to disperse, even though she has reasonable grounds for believing that a breach of the peace will occur.

E. Although the inspector does have common law powers, they only empower her to arrest the demonstrators and not to order them to disperse.

A

C is correct.

As the senior police officer present at the scene, the inspector does have the power under s 14(1) POA 1986 to impose conditions on the protest if she reasonably believes that the assembly will result in serious public disorder, serious damage to property, or serious disruption to the life of the community: the noise generated may seriously disrupt the activities of an organisation in the vicinity or cause a significant impact on people nearby either intimidating or harassing them or causing them alarm or distress; or the purpose of the organisers is to intimidate others with a view to compelling them not to do something which they have a right to do, or to do something which they have a right not to.

However, based on Police v Reid, it seems unlikely that the organisers have intimidatory purpose, and the protest seems unlikely to lead to serious disruption, any disruption to workers’ journey home being likely to be considered fairly minor.

There is also no evidence that the noise generated will have a significant impact on the workers. For those reasons, option A is wrong.

The police do have common law powers to prevent a breach of the peace that can be used to disperse meetings, so option D is wrong.

Option B is wrong because on the facts there are no grounds for apprehending a breach of the peace as defined in R v Howell, as the conduct of protestors seems unlikely to result in violence.

Option E is wrong because the common law powers to prevent a breach of the peace are not limited to arrest but can include a direction to disperse.

85
Q

A woman is taking part in a march through the high street of a town in England. The march is protesting against plans that the Government published two days ago to build a new prison in the town. The march was scheduled to take place during the visit of the government minister responsible for prisons to the town which was only announced yesterday. The woman learnt about the march from a leaflet that was put through the letter box of her home. She has been arrested for taking part in an illegal procession.

Is the woman guilty of an offence?

A. Yes, because the organisers of the march did not give six clear days’ notice of the march. The march is therefore illegal.

B. Yes, because a march during the controversial visit of a government minister is likely to cause serious disruption to the life of the community.

C. No, because although the organisers have committed an offence by failing to give six clear days’ notice, the march itself is not illegal.

D. No, because as it was not reasonably practicable to give advance notice of the march, the organisers have not committed an offence and in any event the march itself is not illegal.

E. No. Although the organisers have committed an offence by failing to give six clear days’ notice, she has not committed an offence as she had no knowledge of this omission.

A

Option D is correct.

Under s 11(1) POA 1986 organisers of a public procession (march) must give the police six clear days’ notice if it is for any of the purposes specified in the section. Protesting against a new prison comes within these purposes.

However, if it is not reasonably practicable to give any advance notice of the procession, there is no duty to give the notice. Here, the march seems to be an immediate response to the announcement of the new prison and coincides with the visit of the responsible government minister. If the organisers had given six clear days’ notice, they would have missed the minister’s visit.

Option D is thus a better answer than option C, as option C sets out what the position would have been had the duty to give notice applied. Option E is wrong as the marches remain legal even when the organisers should have given notice; the knowledge of the marchers whether notice has been given is irrelevant.

Option A is wrong because, as stated above, the march remains legal even if a notice should have been given.

As regards option B, while the prospect of serious disruption to the life of the community may give the police grounds for imposing conditions on the march, it does not render the march itself illegal.

86
Q

A group of 15 anti-vaxxers are demonstrating outside a health clinic. Several people who are attending the clinic to receive vaccinations report feeling uncomfortable as they walk into the clinic past the protestors. A police constable attends the demonstration and tells the anti-vaxxers that they must move their protest to the next street 50m away. The anti-vaxxers refuse to comply with the police constable’s direction. The police constable calls for back up and the protestors are arrested.

Have the police acted lawfully in arresting the anti-vaxxers?

A. Yes because the anti-vaxxers were intimidating those attending the clinic.

B. Yes because the anti-vaxxers were committing a breach of the peace.

C. No because the demonstration by the anti-vaxxers did not constitute a public assembly.

D. No because the police constable is not a senior police officer and so did not have authority to impose directions.

E. No because the police constable had no grounds to justify imposing the condition.

A

The correct answer is option E as the police constable did not have grounds for imposing the condition.

There is no evidence to give rise to a reasonable belief that the demonstration will result in serious public disorder, serious damage to property, serious disruption to the life of the community or that the purpose of the anti-vaxxers is to intimidate those attending the clinic. The protest is silent so there is no noise generated by it.

Option A is wrong as causing discomfort is insufficient to amount to intimidation (Police v Reid).

Option B is wrong as the anti-vaxxers are not committing a breach of the peace which would require ‘harm to be done or likely to be done to a person or in his presence to his property’ or for them to fear harm through an unlawful disturbance (R v Howell). There is no evidence of such harm on the facts.

Option C is wrong as the demonstration does constitute a public assembly – it consists of two or more people in a public place.

Option D is wrong as the police constable is the only, and therefore the most senior, police officer present at the scene.

87
Q

A local authority has issued a notice prohibiting trespassory assemblies within the vicinity of a well-known landmark. A group of about 25 protestors have, with the permission of the farmer who owns the land, gathered on farm land within the area covered by the notice and within the time frame specified in it. The farmer said they could remain on the land as long as they did not camp on it. The police arrested the protestors for taking part in a trespassory assembly after they put up tents on it.

Which of the following best describes whether the protestors are guilty of an offence relating to trespassory assemblies?

A. They are guilty because they have taken part in an assembly in the area covered by the notice.

B. They are guilty because they have taken part in an assembly on private land in the area covered by the notice.

C. They are guilty because they put up tents, breaking the terms of the permission given by the farmer.

D. They are not guilty because the farmer gave them permission to be on the land, so they are not trespassers.

E. They are not guilty because the farmer gave them permission to be on the land, and they have not threatened to breach the peace.

A

C is correct.

Under s 14A the local authority has the power, with the Home Secretary’s consent, to prohibit an assembly likely to be held without the permission of the occupier of the land or to conduct itself in such a way as to exceed the limits of any permission granted by the occupier.

As long as the protestors remain within the limits of the farmer’s permission, the assembly is not trespassory. However, once they put up tents they have exceeded the limits of the permission.

Options A and B are wrong as they define the type of land covered by a prohibition notice too widely.

Options D and E are wrong because the protestors have exceeded the limits of the farmer’s permission.

88
Q

A farmer is participating in a trial of genetically modified wheat on his farm. A group of 25 anti-GM protestors have gathered on the farm in an empty field next to the farm shop. The farmer has called the police saying he fears visitors to the farm shop will be discouraged from buying his produce by the protestors. The police attend and tell the protestors that they must disperse within the next 15 minutes or they will be arrested.

Are the actions of the police lawful?

A. Yes, because the demonstration is being held without the permission of the landowner.

B. Yes, because the police gave the protestors 15 minutes to disperse so their actions are proportionate.

C. Yes, because the demonstration amounts to a trespassory assembly since it consists of more than 20 people.

D. No, because the police do not reasonably fear that a breach of the peace will occur.

E. No, because the demonstration will not result in serious disruption to the life of the community.

A

D is the correct answer as in these circumstances the police will only be able to disperse the demonstrators if they reasonably fear a breach of the peace. The farmer says he fears that visitors to the shop will be discouraged from buying his produce, but this does not amount to fear of harm sufficient for a breach of the peace.

Option A is wrong as the police cannot disperse protestors from private land unless a crime is being committed or there is a breach of the peace. Neither is taking place on the facts.

Option B is wrong. If the police had power to disperse the protestors then their actions would probably be proportionate, but they have no such power.

Option C is wrong since to be a trespassory assembly the demonstration would need to result in serious disruption to the life of the community or significant damage to the land, building or monument which is of historical, archaeological or scientific importance. In addition, the local authority would need to make a banning order.

Option E is wrong since the demonstration is not a public assembly as it is not taking place in a public place as the demonstrators are in a field next to the farm shop.

89
Q

Several members of the public organised a march to protest about a play being performed in their local theatre. They led a group of protestors down the local high street and into the theatre. The protestors were careful not to block the path of members of the public arriving to see the evening performance of the play. The police were called and immediately arrested those at the front of the procession. They were subsequently charged with a public order offence in relation to the procession.

Which of the following might provide the protestors with a defence?

A. A public procession is only ‘public’ if it takes place outside.

B. A public procession only breaches public order if it involves violence or threats of violence.

C. The arrest of the protestors was unlawful because they were no longer on the street when they were detained.

D. The theatre was a privately owned space and therefore not subject to public order provisions.

E. The arrest was unlawful as there had been no conditions imposed upon the protestors which they had then breached.

A

E is correct because an arrest can only be made if conditions have been imposed under s12 or s14 of the POA 1986 and those conditions have been breached. There is no mention of conditions having been imposed in this case.

A is wrong because a public order offence can be committed inside as well as outside provided that it takes place in a public place as defined by s16 POA 1986.

B is wrong because violence or threats of violence are not necessary for a public order offence to be committed.

C is wrong because a theatre falls within the definition of a public place as defined by s16 POA 1986.

D is wrong because s16 POA 1986 makes it clear that a public order offence can be committed in a privately owned space if it is a space to which the public has access on payment or otherwise.

90
Q

The Anti-Fur Alliance (‘AFA’) are planning a protest demonstration on the pavement outside a store which sells fur products. The police have received intelligence that large numbers of members of the Freedom in Fashion Group are planning to attend the demonstration and serious violence is likely to break out as it has done at similar protests in the past. The chief constable believes that due to the number of protestors involved imposing conditions would be ineffective. Instead the Chief Constable would like to ban AFA’s demonstration to prevent any serious violence.

Does the Chief Constable have power to apply to the local authority to ban the AFA’s demonstration?

A. Yes, because the Chief Constable reasonably believes that imposing conditions will not prevent AFA’s demonstration from resulting in serious public disorder.

B. Yes, because the Chief Constable reasonably believes that AFA’s demonstration may result in serious public disorder.

C. Yes, because the Chief Constable reasonably believes that AFA’s demonstration will result in serious public disorder.

D. No, because the Chief Constable has no power to ban AFA’s demonstration in advance.

E. No, because a ban would be disproportionate. The Chief Constable could impose conditions as to the number who could attend the demonstration.

A

The correct answer is Option D as the police have no power to ban a static demonstration in advance.

Option A is wrong as it correctly describes when the police can apply to ban public processions rather than static demonstrations.

Option B is wrong as it relates to the power to impose conditions, but it misstates this power which only applies if the senior police officer reasonably believes the demonstration will result in (amongst other things) serious public disorder.

Option C is wrong as it states the power to impose conditions, not to ban.

Option E is wrong as it states the wrong reason why the Chief Constable cannot ban the demonstration which is because they have no legal power to do so, not because to do so would be disproportionate.

91
Q

A climate change protestor plans to march through the town. She gives the police 7 days’ notice of the proposed march. The police believe the number who will attend is likely to be small, but details of the march are widely shared on social media and several thousand protestors attend. Traffic comes to a standstill and people are unable to get through. Several motorists approach the inspector in charge of policing the march and say that they are trying to get to the airport on the other side of town and will miss their flights if they are delayed much longer. As it is clear that it will take some considerable time for the protestors to make their way through the town, the inspector tells the climate change protestor that she must divert the march away from the main road and into the local park.

Which of the following best explains whether the inspector has acted lawfully in telling the climate change protestor that she must divert the march away from the main road and into the local park?

A. The inspector has acted lawfully because they reasonably believe that the march will result in serious disruption to the life of the community.

B. The inspector has acted lawfully because they reasonably believe that the march will result in serious traffic disruption.

C. The inspector has acted lawfully because they reasonably believe that the march will result in serious public disorder.

D. The inspector has acted unlawfully because such directions can only be given by the Chief Constable.

E. The inspector has acted unlawfully because, as the march is now in progress, the police can only use their powers to prevent a breach of the peace and no breach of the peace is taking place or is imminent.

A

A is the correct answer.

The inspector, as the senior police officer present on the scene, may give such directions as appear necessary to prevent serious disruption to the life of the community. The fact that traffic has come to a standstill and will take a considerable time to clear meaning that people may miss their flights is likely to amount to serious disruption to the life of the community. It will amount to a hindrance that is more than minor to the carrying out of day-to-day activities (including in particular the making of a journey).

B is wrong as serious traffic disruption is not one of the criteria mentioned in the Public Order Act 1986 so that, whilst serious traffic disruption may also amount to serious disruption to the life of the community, this is not the best answer.

C is wrong as there is no suggestion that the march will result in serious public disorder.

D is wrong as, although directions prior to the march taking place can only be given by the Chief Constable, once the march is in progress such directions can be given by the senior police officer on the scene.

E is wrong as the police still have power to give directions under the Public Order Act when the march is in progress.